SlideShare una empresa de Scribd logo
1 de 114
Descargar para leer sin conexión
1
GMAT Verbal Study Guide
Table of Contents
Chapter 1 Reading Comprehension ...............................................................................4
Section 1: One Principle .............................................................................................. 5
Section 2: Two Styles .................................................................................................. 8
1. Presentation....................................................................................................... 8
2. Argumentation .................................................................................................. 9
3. Organizational Structure ..................................................................................11
Section 3: Three Subjects........................................................................................... 14
1. Natural Science............................................................................................... 14
2. Social Science ................................................................................................. 15
3. Business Subject ............................................................................................. 17
Section 4: Four-step Process of Reading ................................................................... 20
1. Analyze the first paragraph............................................................................. 20
2. Skim the passage and get the author's main point........................................... 21
3. Diagram the organization of the passage........................................................ 23
4. Tackle the questions and correspondently refer to the passage. ..................... 26
Section 5: Five Types of Questions ........................................................................... 29
1. Main Idea Question......................................................................................... 30
2. Recall Question............................................................................................... 35
3. Inference Questions ........................................................................................ 37
4. Critical Reasoning Question ........................................................................... 39
5. Difficult-to-locate Question............................................................................ 41
Section 6: Six test points............................................................................................ 43
1. Comparison..................................................................................................... 43
2. Example & Listing.......................................................................................... 43
3. People, Date & Place ...................................................................................... 46
4. Words of Attitude and Transition ................................................................... 47
5. Counter-evidence Indicators ........................................................................... 49
6. Special Punctuation......................................................................................... 51
Chapter 2 Sentence Correction ....................................................................................53
Introduction................................................................................................................ 53
2
Three-step method ..................................................................................................... 54
Section 1: Subject-Verb Agreement .......................................................................... 56
Section 2: Verb Time Sequences ............................................................................... 57
Section 3: Modification.............................................................................................. 58
A. Adjective or adverb as a modifier.................................................................. 58
B. Clause as a modifier ....................................................................................... 58
C. A long phrase as a modifier............................................................................ 59
D. Appositive as a modifier ................................................................................ 60
Section 4: Parallelism ................................................................................................ 61
Section 5: Pronoun..................................................................................................... 62
Section 6: Comparisons ............................................................................................. 63
1. Quality Comparison........................................................................................ 63
2. Quantity Comparison...................................................................................... 64
3. Analogy........................................................................................................... 64
Section 7: Choice of Word......................................................................................... 66
Section 8: Idioms ....................................................................................................... 67
Section 9: Sentence Structure .................................................................................... 72
Section 10: Subjunctive Mood................................................................................... 74
Section 11: Ambiguity ............................................................................................... 75
Section 12: Redundancy ............................................................................................ 76
Section 13: Awkward................................................................................................. 77
Section 14: Logicality................................................................................................ 78
Chapter 3 Critical Reasoning.......................................................................................80
Section 1: Introduction to Critical Reasoning............................................................ 81
1. One Definition: Argument .............................................................................. 81
2. Four elements of an argument......................................................................... 82
3. Seven Common Fallacies................................................................................ 84
4. Three-element Rule......................................................................................... 86
5. Two Traps ....................................................................................................... 87
6. Five Answer Choices ...................................................................................... 88
Section 2: Six Types of Argument............................................................................. 90
1. Deductive Argument....................................................................................... 90
2. Generalization................................................................................................. 94
3. Analogy........................................................................................................... 95
4. Causal Reasoning............................................................................................ 96
5. Finding Assumption........................................................................................ 97
6. Business Thinking........................................................................................... 99
Section 3: Eight Types of Question ......................................................................... 100
1. Inference Question........................................................................................ 100
2. Assumption Question.................................................................................... 102
3. Strengthen Question...................................................................................... 104
3
4. Weaken Question.......................................................................................... 106
5. Paradox Question.......................................................................................... 108
6. Reasoning Question ...................................................................................... 109
7. Complete Question.........................................................................................110
8. Boldface Question..........................................................................................112
4
Chapter 1 Reading Comprehension
Reading Comprehension on the test day
On the test day, you will expect to see three or four reading passages, each followed with three or
four questions. The passages presented depend on how well you are performing on the test.
However, the questions presented for the same passage do not depend on your performance. In
other word, after you are assigned a reading passage, the next question presented for the same
passage will not base on your performance on the previous question.
Why Reading Comprehension is a nightmare to most students?
Most people find the reading compression difficult to prepare because the subject matter is
unfamiliar and could be anything. In order to make sure that nobody can take advantages on a
particular subject, the test-maker takes every effort to diversify the subjects of the three or four
passages on your test day. As a result, obscure subject matter is chosen so that you will be tested,
not on your knowledge of a particular subject, but the test-taking skills.
In addition, the reading passage is not created like the one we see on magazine, newspaper, or
textbook. Rather, it uses a highly compressed style. Subjects of the passages are generally
excerpted from academic articles that were published tens of years ago. Usually the chosen
article is heavily edited until it is cut down to about 300 words, about one-third its original length.
Though it is difficult to read, the reading techniques introduced in the following passage will help
you pick up the right answer even without understanding the reading passage.
Chapter Preview
In order to make it easier for you to prepare for GMAT, we have developed an interesting course
for Reading Comprehension. You will find this chapter all in number, as the section number
suggests. We hope this would help you learn the test prep strategies.
Section 1: One Principle
Section 2: Two Writing Styles
Section 3: Three Subjects
Section 4: Four-step Procedure for Attacking a Passage
Section 5: Five Types of Question
Section 6: Six Test Points
5
Section 1: One Principle
Directions: The questions in this group are based on the content of a passage. After reading the
passage, choose the best answer to each question. Answer all questions following the
passage on the basis of what is stated or implied in the passage
On the test day, you will see the above direction on computer screen. Most students disregard
this instruction since it appears in every test. However, it introduces a basic principle you should
follow in answering a reading comprehension question. When answering questions, you must
refer each of them to some place in the passage. Don't rely on memory, since too many traps are
used with these questions.
Also, don't base on your daily life experiences or college knowledge. Remember, GMAT doesn't
test any specific knowledge on business or other functions. Image if a question is based on some
specific knowledge, then those with broad knowledge can take advantages. This definitely
violates the rule of GMAT. The test-maker often fools you by creating stuff choices that contain
reasonable statement based on basic knowledge or your life experience, not on the passage. If
you find an answer choice contains the widely known reasoning or statement on the test day,
eliminate those choices with hesitation.
Let's look at a sample passage that discusses why the Indian software vendors perform better
than their counterparts in China.
Indian firms have achieved the highest levels of efficiency in the world software outsourcing
industry. Some researchers have assumed that Indian firms use the same programming
languages and techniques as Chinese firms but have benefited from their familiarity with
English, the language used to write software code. However, if this were true, then one
would expect software vendors in Hong Kong, where most people speak English, to perform
not worse than do Indian vendors. However, this is obviously not the case.
Other researchers link high Indian productivity to higher levels of human resource investment
per engineer. But a historical perspective leads to a different conclusion. When the two top
Indian vendors matched and then doubled Chinese productivity levels in the mid-eighties,
human resource investment per employee was comparable to that of Chinese vendors.
Furthermore, by the late eighties, the amount of fixed assets required to develop one
software package was roughly equivalent in India and in the China. Since human resource
investment was not higher in India, it had to be other factors that led to higher productivity.
A more fruitful explanation may lie with Indian strategic approach in outsourcing. Indian
software vendors did not simply seek outsourced contract more effectively: they made
aggressive strategic in outsourcing. For instance, most software firms of India were initially
set up to outsource the contract in western countries, such as United States. By contrary,
6
most Chinese firms seem to position their business in China, a promising yet
under-developed market. However, rampant piracy in China took almost 90 percents of
potential market, making it impossible for most Chinese firms to obtain sufficient
compensation for the investment on development and research, let alone thrive in
competitive environment.
Now, let's look at a sample question:
Which of the following statements concerning the productivity levels of engineers can be
inferred from the passage?
(A) Prior to the 1980’s, the productivity levels of the top Indian software firms were exceeded
by those of Chinese software firms.
(B) The official language of a country has a large effect on the productivity levels of its software
developers.
(C) During the late 1980’s and early 1990’s, productivity levels were comparable in China and
India.
(D) The greater the number of engineers that a software firm has, the higher a firm’s
productivity level.
(E) The amount of human resource investment made by software developers in their firms
determines the level of productivity.
If you do not refer to the original passage, you may pick up B. For test-takers who have some
backgrounds in computer, it is obvious that being familiar with English will gain some advantage
in writing program code. However, the correct answer is C.
In conclusion, the directions can run out of your eyes on the test day, but should be rooted deeply
in your heart at the beginning of your test preparation.
Trap: Some choices just repeat the same words or phrases that you read in the passage. Keep
alarm to these choices since in most cases, they are incorrect.
Here is an example:
The fact that reducing price can generate a competitive advantage for a company does not
mean that every reduction in price will create such an advantage. Price reduction, like
improvement in service, must be balanced against other types of efforts on the basis of direct,
tangible benefits such as increased revenues. If a company is already effectively on a par
with its competitors because it provides product at an acceptable price and keeps customers
7
from leaving at an unacceptable rate, then reduction in price may not be effective, since price
is not necessarily the deciding factor for any customer in any situation.
This truth was not apparent to managers of one operating system software vendor, which
failed to improve its competitive position despite its attempt to reduce price. The software
managers did not recognize the level of customer inertia that arises from the inconvenience
of switching operating system. Nor did they analyze their reduction in price to determine
whether it would attract new customers by producing a new standard of price that would
excite customers or by proving difficult for competitors to copy.
Sample question
According to the passage, reduction in price are comparable to improvement in service in
terms of the
(A) tangibility of the benefits that they tend to confer
(B) increased revenues that they ultimately produce
(C) basis on which they need to be weighed
(D) insufficient analysis that managers devote to them
(E) degree of competitive advantage that they are likely to provide
To answer this question, first locate the question to the second sentence of the passage. "Price
reduction, like improvement in service, must be balanced against other types of efforts on the
basis of direct, tangible benefits such as increased revenues." Now, go back to answer choices.
Choice D and E are irrelevant to the original sentence, so eliminate them. Then, look at the choice
A and B, both of them repeat the original sentences.
(A) tangibility of the benefits that they tend to confer
(B) increased revenues that they ultimately produce
(C) basis on which they need to be weighed
Both A and B seem to be correct. However, reduction in price is comparable to that of
improvement in service in term of the basis on direct and tangible benefits, not on the tangibility or
specific benefits of increased revenues. So neither A nor B is correct. Choice B does not repeat
the same words, but address the basis for comparison. Therefore, C is the correct answer.
8
Section 2: Two Styles
There is an endless number of writing techniques that authors use to present their ideas.
However, there are only two writing styles used in a GMAT reading passage: presentation and
argumentation.
1. Presentation
This technique is to present an idea that the author will agree or at least partially agree. The
author strengthens his position by citing relevant evidences, each related to other in a highly
structured manner. We call this style of writing as presentation. Sometimes, the author sometime
may intentionally contrast his position with an opposing view. But most often the author is just
anticipating an objection, he will soon refute it.
Here is a sample passage in presentation.
China as a nation faces two major financial problems. First, eighty-four percent of
state-owned enterprises do not generate profit. Government failed to collect money from
such business. Rather, it has to appropriate substantial funds to these enterprises in order to
prevent them from going bankrupt and thus resulting in high unemployment rate. Second,
203 million of civilians in countryside will not be able to gain pension after they retire due to
the limited budget of government.
I would like to make an outrageous suggestion that would at one stroke generate finance
earnings and provide funds for civilians’ retirement. I would propose that government sells its
holdings in state-owned enterprises on the open market. Such sales would provide
substantial funds for village civilian’s pension. At the same, they could cut down financial
burden on these state-owned enterprises.
You might object that government would be deprived of the opportunity to share its
enterprise’s profit if someday they make money. I agree. Sell holdings of enterprises that
would never generate profit. But, you might reply, every enterprise that competes on the
market has potential. Here we part company. Theoretically, you may be correct in claiming
that every enterprise has the potential to make money. Practically, you are wrong.
I refer to the thousands of state-owned enterprises that are not likely to make money. These
companies are 100 percent held by the nation as a whole. Government officials are
appointed as the chairman, CEO and president. The management was not responsible for
the public interest, but for the nation as a whole. If there is no significant loss in business,
they will soon be promoted back to the higher level position in government. If their
companies perform great, these executives receive direct money compensation. However,
9
their salary, when combined with such compensation, will be far below that of their
counterpart in private company.
It would be unrealistic to suggest that village civilians would have sufficient funds if
government’s shares were sold on the open market. But the demand for compensating the
state-own enterprises would be substantially reduced.
The author developed the above passage by first pointing out a problem, suggesting a solution,
anticipating counter-position, illustrating an example, refuting a second solution, and further
anticipating possible objections. Obviously, this writing technique is presentation.
2. Argumentation
The second writing style is argumentation. This technique has a number of variations, but the
most common and direct is to develop two to three ideas and then point out why one is better than
the other or just simply refute all of them and developed the author's own idea.
Some common tip-off sentences to this method of analysis are:
z It was traditionally assumed...
z It was once believed...
z It was frequently assumed ..
z It was universally accepted..
z Many scientists have argued...
The passage that discusses Indian and Chinese software firms represents a typical
argumentation. At the beginning, the author presented a phenomenon and gave an explanation,
but refuted that explanation immediately.
Indian firms have achieved the highest levels of efficiency in the world software outsourcing
industry. Some researchers have assumed that Indian firms use the same programming
languages and techniques as Chinese firms but have benefited from their familiarity with
English, the language used to write software code. However, if this were true, then one
would expect software vendors in Hong Kong, where most people speak English, to perform
not worse than do Indian vendors. However, this is obviously not the case.
Then, the second explanation was introduced, but was denied again in the same paragraph.
Other researchers link high Indian productivity to higher levels of human resource investment
per engineer. But a historical perspective leads to a different conclusion. When the two top
Indian vendors matched and then doubled Chinese productivity levels in the mid-eighties,
human resource investment per employee was comparable to that of Chinese vendors.
10
Furthermore, by the late eighties, the amount of fixed assets required to develop one
software package was roughly equivalent in India and in the China. Since human resource
investment was not higher in India, it had to be other factors that led to higher productivity.
Finally, a more fruitful one is presented. The author used the remaining passage try to argue that
this explanation is the correct one.
A more fruitful explanation may lie with Indian strategic approach in outsourcing. Indian
software vendors did not simply seek outsourced contract more effectively: they made
aggressive strategic in outsourcing. For instance, most software firms of India were initially
set up to outsource the contract in western countries, such as United States. By contrary,
most Chinese firms seem to position their business in China, a promising yet
under-developed market. However, rampant piracy in China took almost 90 percents of
potential market, making it impossible for most Chinese firms to obtain sufficient
compensation for the investment on development and research, let alone thrive in
competitive environment.
Why bother to identify the writing style?
Be familiar with the author's writing techniques can help you diagram the mental road map of a
passage, identify the author's intention to cite an evidence, main idea of a passage, and most
importantly, pick up the right choice quickly and decisively. Let’s go back the passage that talks
about whether price reduction can generate a competitive advantage.
The fact that reducing price can generate a competitive advantage for a company does not
mean that every reduction in price will create such an advantage. Price reduction, like
improvement in service, must be balanced against other types of efforts on the basis of direct,
tangible benefits such as increased revenues. If a company is already effectively on a par
with its competitors because it provides product at an acceptable price and keeps customers
from leaving at an unacceptable rate, then reduction in price may not be effective, since price
is not necessarily the deciding factor for any customer in any situation.
This truth was not apparent to managers of one operating system software vendor, which
failed to improve its competitive position despite its attempt to reduce price. The software
managers did not recognize the level of customer inertia that arises from the inconvenience
of switching operating system. Nor did they analyze their reduction in price to determine
whether it would attract new customers by producing a new standard of price that would
excite customers or by proving difficult for competitors to copy.
In the above passage, the author did not try to present his own position (presentation). If any, the
position is that he does not agree with the fact that reduction in price can generate competitive
11
advantage for a company. In fact, the speaker here argued against a popular point of view by
reasoning and examples (argumentation).
Let's look at a sample question to see how to pick up a right choice on the basis of writing styles.
The primary purpose of the passage is to
(A) contrast possible outcomes of a type of business strategy
(B) suggest more careful evaluation of a type of business strategy
(C) illustrate various ways in which a type of business strategy could fail to enhance revenues
(D) trace the general problems of a company to a certain type of business strategy
(E) criticize the way in which managers tend to analyze the costs and benefits of business
strategies
This question asks you to summarize the passage's central idea. Which of the five choices is
correct? Based on the verbs initiating the five choices, you can eliminate three of them:
(A) incorrect. To contrast is to compare several things, but not to agree or disagree.
(C) incorrect. To illustrate is to give example, not to agree or disagree.
(D) incorrect. To trace is to track, not to agree or disagree.
Choice E began with argumental word criticize, but isn't the correct choice because it addresses
the detail. Therefore, B is the right answer: to argue that superior service does not generate
competitive advantage is to suggest more careful evaluation of a type of business strategy (price
reduction).
3. Organizational Structure
There are two major patterns that the test-maker uses to reach a conclusion: general-to-specific
and specific-to-general. Become familiar with these writing patterns can help you identify the main
idea of a passage.
A. General-to-Specific Structure
This structure is widely used in GMAT reading passage. The test-writer first makes a general
argument, and then supports it using a series of specific examples or reasoning, and finally
summaries by reclaiming his general argument.
Here is the structure:
z General claim, followed by
z first evidence or reasoning
z second evidence or reasoning
12
z more evidence or reasoning
Let's look at a passage of this structure:
The fact that reducing price can generate a competitive advantage for a company does not
mean that every reduction in price will create such an advantage. Price reduction, like
improvement in service, must be balanced against other types of efforts on the basis of direct,
tangible benefits such as increased revenues. If a company is already effectively on a par
with its competitors because it provides product at an acceptable price and keeps customers
from leaving at an unacceptable rate, then reduction in price may not be effective, since price
is not necessarily the deciding factor for any customer in any situation.
This truth was not apparent to managers of one operating system software vendor, which
failed to improve its competitive position despite its attempt to reduce price. The software
managers did not recognize the level of customer inertia that arises from the inconvenience
of switching operating system. Nor did they analyze their reduction in price to determine
whether it would attract new customers by producing a new standard of price that would
excite customers or by proving difficult for competitors to copy.
Here, the author presents his opinion at the beginning of the passage: reduction in price does not
necessarily generate a competitive advantage. To support his idea, the author first made
reasoning by comparing service improvement and price reduction. Then, in the second paragraph,
the author used an example within operating system software industry to further address that
reducing price did not improve competitive position.
B. Specific-to-General Structure
Contrast to the general-to-specific structure, the specific-to-general first presents a group of
examples or reasoning and finally draw a conclusion.
Here is the structure:
z first example or reasoning
z second example or reasoning
z more example or reasoning
z Conclusion
The passage that discusses Indian software vendors was written in argumentation, and
represents a typical passage in specific-to-general structure.
Indian firms have achieved the highest levels of efficiency in the world software outsourcing
industry. Some researchers have assumed that Indian firms use the same programming
languages and techniques as Chinese firms but have benefited from their familiarity with
13
English, the language used to write software code. However, if this were true, then one
would expect software vendors in Hong Kong, where most people speak English, to perform
not worse than do Indian vendors. However, this is obviously not the case.
Other researchers link high Indian productivity to higher levels of human resource investment
per engineer. But a historical perspective leads to a different conclusion. When the two top
Indian vendors matched and then doubled Chinese productivity levels in the mid-eighties,
human resource investment per employee was comparable to that of Chinese vendors.
Furthermore, by the late eighties, the amount of fixed assets required to develop one
software package was roughly equivalent in India and in the China. Since human resource
investment was not higher in India, it had to be other factors that led to higher productivity.
A more fruitful explanation may lie with Indian strategic approach in outsourcing. Indian
software vendors did not simply seek outsourced contract more effectively: they made
aggressive strategic in outsourcing. For instance, most software firms of India were initially
set up to outsource the contract in western countries, such as United States. By contrary,
most Chinese firms seem to position their business in China, a promising yet
under-developed market. However, rampant piracy in China took almost 90 percents of
potential market, making it impossible for most Chinese firms to obtain sufficient
compensation for the investment on development and research, let alone thrive in
competitive environment.
In the above passage, the author gave an explanation to a particular event, but refuted it soon,
until it came with a convincing one – the conclusion.
14
Section 3: Three Subjects
Like writing techniques, GMAT subjects may vary significantly. The author may present how
caffeine activates human behavior, discuss what causes Japanese auto companies to perform
better than those in USA, or explain the union's effort to organize the employees in public sectors.
Various as the subjects may be, there are only three major subjects that a GMAT passage may
be discussing about: natural science, social science, and business. As the name indicates,
natural science topic includes biology, chemistry, geology, and archeology; social science
includes art, literature, and civil rights; business includes marketing, advertising, management,
and economics.
1. Natural Science
Characteristics
Most test-takers find the natural science difficult to read and beyond their knowledge base. If you
try to figure out what they are really talking about, in most cases you will fail, because reading
passages in this kind of subject are filled of nomenclatures and jargons. The good news, however,
is that the sentences are always written in a simple syntax and the questions to be answered after
reading passage are typical of the recalled questions. That means, when you successfully locate
the "key words", you will find it easy to get the right answer.
Strategy
Don't memorize the details or try to figure out the author's reasoning. Skim the passage, and get
its central idea as well as organizational structure.
The following passage is about natural science topic. This passage is a little difficult to
understand, but the following questions are much easier to answer.
Sample Passage
The cutting-edge science is ringing alarm bells. Avian flu virus picked up by pigs can swap
genetic materials with another flu virus already in the pig and become a new, hitherto
unknown flu virus for which no person, no animal has preexisting immunity. The kind of virus
causes a pandemic because it spreads from human to human.
If you took a peek into history, it turns out that previous influenza pandemics have similar
scenarios. The greatest influenza pandemic in 1918 caused more than 20 million deaths of
soldiers stationed in France. The last influenza pandemic was in 1968, known as the Hong
Kong flu (H3N2). Thousands of deaths and millions were infected worldwide.
The other examples are the Nipah virus and Japanese Encephalitis virus, which find pigs to
be good hosts. With JE, the virus circulates in the blood of infected pigs. When infected pigs
15
are bitten by Culex mosquitoes, the virus replicates in the mosquito's gut. The next time the
mosquito bites a human, the virus is passed on. The pig doesn't get sick as such. The Nipah
virus causes pneumonia symptoms in pigs. In humans, it causes encephalitis, and humans
catch it only with direct contact with infected pigs. Symptoms range from mild headache to
permanent brain damage, and can be fatal.
It's merely a phenomenon of nature that the pig is the "mixing vessel" for the new germ. But
make no mistake, the pig is not the villain, neither is the chicken. It's actually us, and our
horrible farm practices, outdated agricultural policy and, most of all, reckless disregard of our
ecology and environment. "Hygiene and management can control what eventually happens,"
says Lam. "Good farming practice will prevent serious outbreaks and infection to humans."
Despite knowing that, animal diseases and the possibility of transmission to humans are
becoming quite alarming. Of the 35 new emerging diseases in the last 20 years, more than
70 per cent involved animals.
In fact, what we may have done is unwittingly create the perfect launch pad for an influenza
pandemic that will likely kill large numbers of people across the globe. Although scientists
say it's impossible to predict the odds that the virus will alter its genetic form radically enough
to start leaping from human to human, the longer H5N1 is out there killing chickens, the
higher the chances are.
Sample Question
Which of the following statement can be inferred from the passage?
(A) New emerging diseases causes more deaths of human than animal.
(B) Animals are the villain for most flues.
(C) Hygiene and management can not control the spread of viruses.
(D) The current bird flu epidemic may be a launch pad for the next influenza pandemic.
(E) The influenza pandemic is always a regional phenomenon.
Which answer is correct? For choice A, the passage did not make any comparison between
deaths of human and deaths of animal. In B, animal is actually not the villain for most flues. Rather,
it is human. Look at the second sentence in the fourth paragraph, “But make no mistake, the pig
is not the villain, neither is the chicken.” For C, “Hygiene and management can control what
eventually happens”(in the middle of fourth paragraph), therefore, C is incorrect. E is also
incorrect. Though most flues discussed in this passage were originated from some areas, the
passage never stated it was a regional phenomenon. In fact, it “will likely kill large numbers of
people across the globe”, as stated at the beginning of last paragraph. The correct answer is D –
the current bird flu epidemic may be a launch pad for the next influenza pandemic, because no
animal has preexisting immunity and it causes a pandemic by spreading from human to human.
16
2. Social Science
Characteristics
Why women's rights experienced a significant improvement during 1860's? How the Pullman
stroke to improve their living condition? Passages in these subjects are easy to read because it
goes as you expect and talks about something around your world. You will find it easy to grasp
the main idea and passage map. In order to get the right answer, however, you need to read
beyond the words, phrases or concepts in the passage. The right answer is always created in a
synthesized way.
Strategy
Be careful in tackling this "social" passage. To answer the later questions is always not as easy
as to understand the passage. The answer choice that contains the exact words or phrases from
the passage is generally not the correct answer. Rather, you need to synthesize several
sentences or make some reasoning before you pick up the right choice. The process is
time-consuming because the social passage is typically long.
Sample Passage
China as a nation faces two major financial problems. First, eighty-four percent of
state-owned enterprises do not generate profit. Government failed to make money from such
business. Rather, it has to appropriate substantial funds to these enterprises in order to
prevent them from going bankrupt and thus resulting in high unemployment rate. Second,
203 million of civilians in countryside will not be able to gain pension after they retire due to
the limited budget of government.
I would like to make an outrageous suggestion that would at one stroke generate finance
earnings and provide funds for civilians’ retirement. I would propose that government sells its
holdings in state-owned enterprises on the open market. Such sales would provide
substantial funds for village civilian’s pension. At the same time, they could cut down
financial burden on these state-owned enterprises.
You might object that government would be deprived of the opportunity to share its
enterprise’s profit if someday they make money. I agree. Sell holdings of enterprises that
would never generate profit. But, you might reply, every enterprise that competes on the
market has potential. Here we part company. Theoretically, you may be correct in claiming
that every enterprise has the potential to make money. Practically, you are wrong.
I refer to the thousands of state-owned enterprises that are not likely to make money. These
companies are 100 percent held by the nation as a whole. Government officials are
appointed as the chairman, CEO and president. The management was not responsible for
17
the public interest, but for the nation as a whole. If there is no significant loss in business,
they will soon be promoted back to the higher level position in government. If their
companies perform great, these executives receive direct money compensation. However,
their salary, when combined with such compensation, will be much less than the amount
they would earn if were in private company.
It would be unrealistic to suggest that village civilians would have sufficient funds if
government’s shares were sold on the open market. But the demand for compensating the
state-own enterprises would be substantially reduced.
Sample Question
According to the passage, executives in a state-owned enterprise are motivated by
(A) direct money compensation
(B) increasing salary
(C) political outlook
(D) share option
(E) social responsibility
The passage mentioned the executives of state-owned enterprises in fourth paragraph; therefore,
we need not to consider other paragraphs when referring to the original passage. Since “…their
salary, when combined with such compensation, will be much less than the amount they
would earn if were in private company” as stated in the last sentence, these executives are not
motivated by financial earnings. If yes, they will transfer to a private company. Therefore, they are
not motivated by direct money compensation, increasing salary, or share option. Rather, they are
concerned on their political outlook. “If there is no significant loss in business, they will soon
be promoted back to the higher level position in government.” Choice C is the correct answer.
For choice E, the passage never discussed the executives’ social responsibility.
18
3. Business Subject
Characteristics
This subject is highly welcomed since most students possess some knowledge or background in
business. But passage of this subject contains the most difficult questions in GMAT Reading
Comprehension. Recall questions are few and you always have to reason before you pick up the
correct choice.
Strategy
Don't rely on your memory even if you become or have been quite familiar with its topics. There
are too many traps here. Make sure you refer to the passage when answering the questions.
Sample Passage
The fact that reducing price can generate a competitive advantage for a company does not
mean that every reduction in price will create such an advantage. Price reduction, like
improvement in service, must be balanced against other types of efforts on the basis of direct,
tangible benefits such as increased revenues. If a company is already effectively on a par
with its competitors because it provides product at an acceptable price and keeps customers
from leaving at an unacceptable rate, then reduction in price may not be effective, since price
is not necessarily the deciding factor for any customer in any situation.
This truth was not apparent to managers of one operating system software vendor, which
failed to improve its competitive position despite its attempt to reduce price. The software
managers did not recognize the level of customer inertia that arises from the inconvenience
of switching operating system. Nor did they analyze their reduction in price to determine
whether it would attract new customers by producing a new standard of price that would
excite customers or by proving difficult for competitors to copy.
Sample Question
The passage suggests which of the following about price charged by an operating system software
vendor prior to its strategy in reducing its price?
(A) It was slightly low to that of the vendor’s competitors.
(B) It threatened to weaken the vendor’s competitive position with respect to other operating
system software vendor
(C) It had already been reduced after having caused damage to the vendor’s reputation in the
past.
(D) It enabled the vendor to retain customers at an acceptable rate
19
(E) It needed to be reduced to attain parity with the software provided by competing vendors.
Here, the question was created in complicated clauses and itself already hard to understand. In
fact, it asks for the situation of the vendor before price reduction. Only D can be inferred from the
passage. The original passage stated that “If a company is already effectively on ….. keeps
customers from leaving at an unacceptable rate…” and “This truth was not apparent to
managers of one operating system software vendor…” That means the vendor was able to
retain customers at an acceptable rate.
20
Section 4: Four-step Process of Reading
In the previous section we summarize three kinds of subject you will encounter in a GMAT
reading passage. Now you will learn the four-step procedure to read a passage in any subject:
1. Analyze the first paragraph.
2. Skim the passage and get some idea of the main idea
3. Identify the purpose of each paragraph and structure of the passage
4. Answer the questions and don't forget to refer to the passage
1. Analyze the first paragraph.
It is essential to carefully read the first paragraph. You will get informed what the passage is
talking about, and even the main idea of the passage. There are two major reasons for you to
carefully read the first paragraph.
Fist of all, the paragraph is the main structural unit of any passage. Every paragraph is needed to
understand the whole passage or answer the question after the passage. Test-maker never
delivers a junk content. It must talk about something that relates to the central idea, and present it
as persuasively as possible. In fact, the first paragraph introduces either the position that the
author will support or the one that he/she will argue against. So, getting familiar with the
introductory paragraph will definitely help you identify the main topic.
Secondly, analyzing the fist paragraph in stead of the whole passage can save you much time. As
I said at the beginning of this chapter, GMAT reading passage is dry and unfamiliar. It is highly
likely that after you read the passage, you get no ideas about what the passage is talking about. If
you go back and reread the whole passage, you will have no sufficient time to answer the
question. Analyze the first paragraph, pay attention to concepts, and then you will find it easy to
understand the subject of passage.
Below is the first paragraph of a GMAT reading passage. Pay attention to concept words.
China as a nation faces two major financial problems. First, eighty-four percent of
state-owned enterprises do not generate profit. Government failed to make money from such
business. Rather, it has to appropriate substantial funds to these enterprises in order to
prevent them from going bankrupt and thus resulting in high unemployment rate. Second,
203 million of civilians in countryside will not be able to gain pension after they retire due to
the limited budget of government.
The first sentence stated that China faces two problems. Then, the author specified these two
problems using a clear and logical structure. Firstly, government did not make money from but
21
input large amount of money to its enterprises. Secondly, government has limited funds for
pension.
Now, let's summarize this paragraph and put it in our own words-- China has two problems:
financial burden and limited funds. Keep these key words (concepts) in mind, and you will find it
easy to understand the remaining passage that we'll present in next step.
2. Skim the passage and get the author's main point
Here are some strategies that will speed your reading and help you identify the author's main
points:
z Focus on the first sentence of each paragraph
The first sentence of a paragraph is always the main point of this paragraph. Why? It confirms to
the formal writing style. If you are a management consultant, you will find it a great advantage to
use a summary at the very beginning of each section. Image when you are presenting a strategy
report which contains hundreds of pages, how could your clients catch your all of them? The only
solution is to use a highly structured presentation, and summarize your idea at the beginning of
each section. In fact, you are also doing like this in the AWA section.
By simply reading the first sentence of each paragraph, you can construct a mental road map of
the passage while not spending significant time.
z Pay attention to the mood words
"Mood" words are those that the author uses to demonstrate his/her position to a particular event,
phenomenon, or point of view. A mood word can be positive or negative. Positive words such as
successfully, correctly and right often illustrate an idea that the author agree. And vis-a-vis, a
negative word indicates an idea that will be weakened in later passage.
The following sentences express the author's position by using positive mood words:
a) Haney's through research provides convincing field evidence that..
b) For many yeas, Benjamin Quarles' seminal account of the participation of African Americans
in the American Revolution has remained the standard work in the field.
c) Roger Rosenblatt's book successfully alters the approach taken by most previous studies.
By contrast, the following mood words are negative.
fail ignore overestimate underestimate
misunderstand misrepresent overlook exaggerate
sound convincingly successfully correctly
22
z Never ignore the counter-evidence indicators
The author uses counter-evidence words not to argue against himself, but concede certain minor
points that may weaken his argument. The counter evidence is finally refuted by further evidence.
You should keep alarm to these words since some students often mistake them as introducing
arguing against a statement.
Following are some of the most common used counter-evidence indicators:
actually despite admittedly except
even though nonetheless nevertheless although
however In spite of do may
OK. Let's go back to the passage talking about national finance. Here are the other five
paragraphs. In order for you to skim the passage using the above three techniques, we
underlined the first sentences of each paragraph, boldfaced the mood words and italicize the
counter-evidence indicators.
I would like to make an outrageous suggestion that would at one stroke generate finance
earnings and provide funds for civilians’ retirement. I would propose that government sells its
holdings in state-owned enterprises on the open market. Such sales would provide
substantial funds for village civilian’s pension. At the same time, they could cut down
financial burden on these state-owned enterprises.
You might object that government would be deprived of the opportunity to share its
enterprise’s profit if someday they make money. I agree. Sell holdings of enterprises that
would never generate profit. But, you might reply, every enterprise that competes on the
market has potential. Here we part company. Theoretically, you may be correct in claiming
that every enterprise has the potential to make money. Practically, you are wrong.
I refer to the thousands of state-owned enterprises that are not likely to make money. These
companies are 100 percent held by the nation as a whole. Government officials are
appointed as the chairman, CEO and president. The management was not responsible for
the public interest, but for the nation as a whole. If there is no significant loss in business,
they will soon be promoted back to the higher level position in government. If their
companies perform great, these executives receive direct money compensation. However,
their salary, when combined with such compensation, will be much less than the amount
they would earn if were in private company.
23
It would be unrealistic to suggest that village civilians would have sufficient funds if
government’s shares were sold on the open market. But the demand for compensating the
state-own enterprises would be substantially reduced.
What is the main idea of the passage? In a word, the author is to present a solution to funding
civilian’s pension while benefiting the state-owned enterprises.
3. Diagram the organization of the passage
You got main idea of each paragraph. Now, it’s time to ask yourself why the author includes them,
what the purpose of each paragraph is, and how each paragraph relates to other. This will help
you diagram the organization of a passage, and locate the details when you answer the
questions.
Pivotal words can help you in diagramming the organization. Pivotal words are signal words or
phrases that would in advance indicate the idea of paragraphs. Below represents the most
frequently used pivotal words or sentences you will see in a reading passage.
Note: A and B represent something, while sb represents somebody.
Introduction
z When it comes to ..., some think ...
z There is a public debate today that ...
z A is a common way of ..., but is it a wise one?
z Recently the problem has been brought into focus.
Presenting Opinion
z Now there is a growing awareness that...
z It is time we explore the truth of ...
z Nowhere in history has the issue been more visible.
Further Presenting Opinion
z ... but that is only part of the history.
z Another equally important aspect is ...
z A is but one of the many effects. Another is ...
z Besides, other reasons are...
Anticipating Objections
z You may reply that.
24
z Admittedly, ..
z It is reasonable to expect...
z It is not surprising that...
Exampling
z For example(instance),...
z ... such as A,B,C and so on (so forth)
z A good case in point is...
z A particular example for this is...
Presenting Reasons
z There are many reasons for ...
z Why .... , for one thing,...
z The answer to this problem involves many factors.
z Any discussion about this problem would inevitably involves ...
z The first reason can be obliviously seen.
z Most people would agree that...
z Some people may neglect that in fact ...
z Others suggest that...
z Part of the explanation is ...
Comparing
z The advantages for A for outweigh the disadvantages of...
z Although A enjoys a distinct advantage ...
z Indeed , A carries much weight than B when sth is concerned.
z A maybe ... , but it suffers from the disadvantage that...
Transitioning
z To understand the truth of ..., it is also important to see...
z A study of ... will make this point clear
Further Anticipating Objections
z Certainly, B has its own advantages, such as...
z I do not deny that A has its own merits.
25
Conclusion
z From what has been discussed above, we may safely draw the conclusion that ...
z In summary, it is wiser ...
z In short...
In step 2, you are assigned to skim the passage and get the main idea. Here, let's identify the
purpose of each paragraph for the archeology passage to better understand the passage.
(First of all, the author presented the problems)
China as a nation faces two major financial problems. First, eighty-four percent of
state-owned enterprises do not generate profit. Government failed to make money from such
business. Rather, it has to appropriate substantial funds to these enterprises in order to
prevent them from going bankrupt and thus resulting in high unemployment rate. Second,
203 million of civilians in countryside will not be able to gain pension after they retire due to
the limited budget of government.
(Then, the author suggested a solution to the problems)
I would like to make an outrageous suggestion that would at one stroke generate finance
earnings and provide funds for civilians’ retirement. I would propose that government sells its
holdings in state-owned enterprises on the open market. Such sales would provide
substantial funds for village civilian’s pension. At the same time, they could cut down
financial burden on these state-owned enterprises.
(Here, the author anticipated a possible objection)
You might object that government would be deprived of the opportunity to share its
enterprise’s profit if someday they make money. I agree. Sell holdings of enterprises that
would never generate profit. But, you might reply, every enterprise that competes on the
market has potential. Here we part company. Theoretically, you may be correct in claiming
that every enterprise has the potential to make money. Practically, you are wrong.
(Then, the author gave an example to deny this objection)
I refer to the thousands of state-owned enterprises that are not likely to make money. These
companies are 100 percent held by the nation as a whole. Government officials are
appointed as the chairman, CEO and president. The management was not responsible for
the public interest, but for the nation as a whole. If there is no significant loss in business,
they will soon be promoted back to the higher level position in government. If their
companies perform great, these executives receive direct money compensation. However,
26
their salary, when combined with such compensation, will be much less than the amount
they would earn if were in private company.
(Finally, the author further anticipated a possible objection)
It would be unrealistic to suggest that village civilians would have sufficient funds if
government’s shares were sold on the open market. But the demand for compensating the
state-own enterprises would be substantially reduced.
Now, you are able to create a mental road map for the whole passage:
Paragraph # 1: introduced two major problems that China faces.
Paragraph # 2: suggested a solution and explained why it is effective.
Paragraph # 3: anticipated a possible objection and denied it soon.
Paragraph # 4: exemplified to argue against a position initiated in the third paragraph.
Paragraph # 5: concluded that his solution is not perfect, but really effective
By making such a road map, I bet you understand this passage quite well.
4. Tackle the questions and correspondently refer to the passage.
Now that you have grasped main idea and the organizational structure of the passage, you are
about to answer the following questions. Again, don’t base on your memory. Always refer to the
original passage before you pick up a choice.
China as a nation faces two major financial problems. First, eighty-four percent of
state-owned enterprises do not generate profit. Government failed to make money from such
business. Rather, it has to appropriate substantial funds to these enterprises in order to
prevent them from going bankrupt and thus resulting in high unemployment rate. Second,
203 million of civilians in countryside will not be able to gain pension after they retire due to
the limited budget of government.
I would like to make an outrageous suggestion that would at one stroke generate finance
earnings and provide funds for civilians’ retirement. I would propose that government sells its
holdings in state-owned enterprises on the open market. Such sales would provide
substantial funds for village civilian’s pension. At the same time, they could cut down
financial burden on these state-owned enterprises.
You might object that government would be deprived of the opportunity to share its
enterprise’s profit if someday they make money. I agree. Sell holdings of enterprises that
would never generate profit. But, you might reply, every enterprise that competes on the
27
market has potential. Here we part company. Theoretically, you may be correct in claiming
that every enterprise has the potential to make money. Practically, you are wrong.
I refer to the thousands of state-owned enterprises that are not likely to make money. These
companies are 100 percent held by the nation as a whole. Government officials are
appointed as the chairman, CEO and president. The management was not responsible for
the public interest, but for the nation as a whole. If there is no significant loss in business,
they will soon be promoted back to the higher level position in government. If their
companies perform great, these executives receive direct money compensation. However,
their salary, when combined with such compensation, will be much less than the amount
they would earn if were in private company.
It would be unrealistic to suggest that village civilians would have sufficient funds if
government’s shares were sold on the open market. But the demand for compensating the
state-own enterprises would be substantially reduced.
1. The primary purpose of the passage is to propose
(A) an alternative to manage government property
(B) a way to relieve government burden while providing funds to village civilians
(C) a way to distinguish state-owned enterprises that make money from those that do no
make money
(D) the governmental approach to evaluate state-owned enterprise’ executives
(E) a new system for national pension system
This question requires you to identify the primary concern of the passage as a whole. The first
paragraph introduces two major problems that China faces. The second paragraph suggests a
solution and explains why it is effective. The third anticipates a possible objection and refutes it
soon. The fourth paragraph illustrates an example to support the author’s argument. In the last
paragraph, the author concludes that his solution is not perfect, but really effective. Therefore, the
correct answer is B.
2. The author implies that all of the following statements about enterprises with which government
holds 100 percent share are true EXCEPT:
(A) A market for government’s share already exists.
(B) Such enterprises seldom generate profit.
(C) There is likely to be a continuing loss of such enterprises.
(D) Government officers are appointed as the executives with such enterprises.
28
(E) If the executives perform poorly, they will be demoted to lower position.
The question requires you to identify the answer choice that CANNOT be inferred from the
passage. Nothing in the passage implies that “the executives will be demoted to lower position if
they perform poorly”. Therefore, the best answer is E. In answering the question that contains
“EXCEPT”, keep alarm not to be fooled by the test maker.
3. The author implies that which of the following would occur if government’s shares were sold on
the open market?
I. The shortage of retirement fund in village would eventually cease completely.
II. Current executives in state-owned enterprises are not motivated to perform better
III. Civilians in countryside would be able to seek sufficient funds from government.
(A) I only
(B) II only
(C) I and II only
(D) II and III only
(E) I, II, and III
This question asks you to identify information that is suggested rather than directly stated in the
passage. To answer it, first look for the location in the passage of the information specified in
answer choice. The last paragraph states that “It would be unrealistic to suggest that village
civilians would have sufficient funds if government’s shares were sold on the open market”,
therefore, I is incorrect. III, which is a repeated of I, is also incorrect. Only II can be inferred from
the original passage, therefore B is the best answer.
29
Section 5: Five Types of Questions
While the techniques introduced in previous four sections speed your reading, this section is
developed to help you pick up the right choice quickly and decisively. In the following passage, we
will discuss the major question types you may encounter in real GMAT test. Generally, there are
only five major types of questions. As you become familiar with the following question types, you
will gain an intuitive sense for the places from which questions are likely to be drawn. Note, the
order in which the questions are asked roughly corresponds to the order in which the main issues
are presented in the passage. Early questions should correspond to information given early in the
passage, and so on.
Of course, there are many other kinds of classification according to different criteria. Here, we
classify, by how we solve reading comprehension questions, into five based on the summary of
thousands of the previous real questions. Let's preview the five question types.
Question Types Preview
1. Main Idea Question
a) Main Topic
b) Tone
c) Structure
d) Exemplifying
2. Recall Question
a) Description
b) Listing
3. Inference Question
4. Critical Reasoning Question
a) Analogy
b) Assumption/Weaken/Strengthen
5. Unable-to-locate Question
30
1. Main Idea Question
There are four sub-types for this kind of question: Main Topic, Tone, Structure, and Exemplifying.
Why should we incorporate them into one type of question? In answering Main Idea Question,
you should understand the organizational structure of the passage, the author tone toward a
particular point of view in the reading passage, the purpose of each paragraph and why a
particular example was illustrated. In other words, if you can determine the main topic of the
passage, you are simultaneously well informed with the structure, the intent of specific example,
and tone toward specific position.
A. Main Topic
Main idea questions test your ability to identify and understand an author's intent in a passage.
The main idea is usually stated in the first or last paragraph. Main idea questions are usually the
first questions asked. Some common main idea questions include:
z The primary purpose of the passage is to…
z Which of the following titles would best describe the content of the passage?
z The passage supplies information that would answer which of the following questions?
z Which of the following is the principal topic of the passage?
z The passage is most probably an excerpt from.
z Which of the following best states the central idea of the passage?
In most cases, main idea questions are easy to solve. In most GMAT passage the author's
primary purpose is to persuade the reader to accept her opinion. Occasionally, it is to describe
something. By determining the relationship of each paragraph, you come up with the main ideal at
the same time. However, the GMAT writers may obscure the correct answer by surrounding it
with close answer choices that stress specifics. Eliminate these choices without hesitation on the
test day.
Trap 1: The main topic will not focus on certain details in the passage. If you encounter the main
ideal question, eliminate the answer choices that describe the details.
Trap 2: Pay special attention to the "repeat" answer. Certain choices may exactly repeat some or
most words of the correct answer, but do not present the central idea, therefore, is not the correct
answer.
B. Tone Question
Tone questions ask you to identify the writer's attitude. Is the writer's feeling toward the subject
positive, negative, or neutral? The following represents some ways of the questions asked.
31
z Which of the following best summarizes the author's evaluation of Bailyn's fourth
proposition?
z The author's attitude toward the culture in most factories is best described as
(A) cautious (B) critical (C) disinterested (D) respectful (E) adulatory
However, if you did not get a feel for the writer's attitude on the first reading, check the mood
words that he chooses. Beware of answer choices that contain extreme emotions. Remember the
passages are taken from academic journals. In the rarefied air of academic circles, strong
emotions are considered inappropriate and sophomoric. The writers want to display opinions that
are considered and reasonable, not spontaneous and off-the-wall. So if an author's tone is
negative, it may be disapproving, not snide or ridiculous. If her tone is positive, it may be
approving, not ecstatic. Or if her tone is neutral, it would be not be disinterested.
C. Organizational Structure
When you can determine the right answer for main topic, generally you have been familiar with
the organizational structure. Every passage is consisted of some paragraphs, and each single
paragraph performs some certain function to the passage as a whole, by presenting, supporting
or refuting the central idea. So, think about the purpose of each paragraph as you read through
the passage.
You may encounter one type of question concerning the main idea or purpose of some certain
paragraph. Some common questions include:
z The last paragraph of the passage performs which of the following functions?
z Which of the following best describes the organization of the second paragraph?
z Which of the following best describes the organization of the passage?
z Which of the following best describes the relation of the first paragraph to the passage as
a whole?
It is relatively easy to solve this kind of question for two reasons. First, however the question may
ask, it is concerning the main idea of the paragraph. If you come up with a question including
"paragraph", it definitely require you to generate that paragraph. Second, paragraph is only some
element of the completed passage. If you can generate the main topic for the whole passage of
three to four paragraphs, why aren't you able to summarize just one paragraph?
D. Exemplifying
The other type of structure question, exemplifying, tests your ability to identify the intention of
author's illustrating of something, some people, or phenomenon. In answering this question, you
need to first locate the example, and then refer to opinion preceding or accompanying the
example. The right answer is the repeat of this opinion.
z In illustrating the example in line 13-16, the author intended to.?
32
z The author referred to the experiment in order to.?
In addition to the above four types, the Main Idea Question may require you to respond in other
ways, such as:
z From what kinds of subject could this passage be excerpted?
z Which of the following topic would be preceding this passage?
This question, however, is similar to the above four, since they are based on your understanding
of the completed passage.
Sample Question #1
The cutting-edge science is ringing alarm bells. Avian flu virus picked up by pigs can swap
genetic materials with another flu virus already in the pig and become a new, hitherto
unknown flu virus for which no person, no animal has preexisting immunity. The kind of virus
causes a pandemic because it spreads from human to human.
If you took a peek into history, it turns out that previous influenza pandemics have similar
scenarios. The greatest influenza pandemic in 1918 caused more than 20 million deaths of
soldiers stationed in France. The last influenza pandemic was in 1968, known as the Hong
Kong flu (H3N2). Thousands of deaths and millions were infected worldwide.
The other examples are the Nipah virus and Japanese Encephalitis virus, which find pigs to
be good hosts. With JE, the virus circulates in the blood of infected pigs. When infected pigs
are bitten by Culex mosquitoes, the virus replicates in the mosquito's gut. The next time the
mosquito bites a human, the virus is passed on. The pig doesn't get sick as such. The Nipah
virus causes pneumonia symptoms in pigs. In humans, it causes encephalitis, and humans
catch it only with direct contact with infected pigs. Symptoms range from mild headache to
permanent brain damage, and can be fatal.
It's merely a phenomenon of nature that the pig is the "mixing vessel" for the new germ. But
make no mistake, the pig is not the villain, neither is the chicken. It's actually us, and our
horrible farm practices, outdated agricultural policy and, most of all, reckless disregard of our
ecology and environment. "Hygiene and management can control what eventually happens,"
says Lam. "Good farming practice will prevent serious outbreaks and infection to humans."
Despite knowing that, animal diseases and the possibility of transmission to humans are
becoming quite alarming. Of the 35 new emerging diseases in the last 20 years, more than
70 per cent involved animals.
In fact, what we may have done is unwittingly create the perfect launch pad for an influenza
pandemic that will likely kill large numbers of people across the globe. Although scientists
say it's impossible to predict the odds that the virus will alter its genetic form radically enough
33
to start leaping from human to human, the longer H5N1 is out there killing chickens, the
higher the chances are.
Which of the following best describes the topic of the passage?
(A) What causes the Nipah virus and Japanese Encephalitis virus to happen?
(B) Does Hong Kong flu originate from pig?
(C) From fowl to pigs to humans?
(D) Is influenza pandemic horrible?
(E) Shall we eat chicken?
This question asks you to find a title for the passage. In other word, it requires you to identify the
primary concern of the passage as a whole. The first paragraph presents a recent virus. The
second and third paragraphs describe similar influenza pandemics in history. The fourth
paragraph concludes who should be responsible for the spread of virus and what human can do
to control. The last paragraph indicates that people stimulated rather than inhibited its
promulgation. We can thus conclude the current virus will also leap to human. Furthermore, the
passage as a whole is to “ring alarm bells”. Therefore, C is the best answer.
Sample Question #2
Indian firms have achieved the highest levels of efficiency in the world software outsourcing
industry. Some researchers have assumed that Indian firms use the same programming
languages and techniques as Chinese firms but have benefited from their familiarity with
English, the language used to write software code. However, if this were true, then one
would expect software vendors in Hong Kong, where most people speak English, to perform
not worse than do Indian vendors. However, this is obviously not the case.
Other researchers link high Indian productivity to higher levels of human resource investment
per engineer. But a historical perspective leads to a different conclusion. When the two top
Indian vendors matched and then doubled Chinese productivity levels in the mid-eighties,
human resource investment per employee was comparable to that of Chinese vendors.
Furthermore, by the late eighties, the amount of fixed assets required to develop one
software package was roughly equivalent in India and in the China. Since human resource
investment was not higher in India, it had to be other factors that led to higher productivity.
A more fruitful explanation may lie with Indian strategic approach in outsourcing. Indian
software vendors did not simply seek outsourced contract more effectively: they made
aggressive strategic in outsourcing. For instance, most software firms of India were initially
set up to outsource the contract in western countries, such as United States. By contrary,
most Chinese firms seem to position their business in China, a promising yet
34
under-developed market. However, rampant piracy in China took almost 90 percents of
potential market, making it impossible for most Chinese firms to obtain sufficient
compensation for the investment on development and research, let alone thrive in
competitive environment.
Which of the following best describes the organization of the first paragraph?
(A) A thesis is presented and supporting examples are provided.
(B) Opposing views are presented, classified, and then reconciled.
(C) A fact is stated, and an explanation is advanced and then refuted.
(D) A theory is proposed, considered, and then amended.
(E) An opinion is presented, qualified, and then reaffirmed.
This question requires you to identify the organizational structure of the first paragraph. In this
paragraph, the author first states a fact that Indian firms achieved the highest efficiency in software
outsourcing. Then, an assumption is presented to explain such phenomenon. However, the author
refuted this explanation soon. Thus, C is the best answer.
35
2. Recall Question
There are two subtypes of recall questions: detail-locating and listing. In the following passage,
we'll discuss one by one.
A. Detail-locating
Locating question is the most common question you will encounter in Reading Comprehension. It
roughly constitutes to 50-60% of total numbers of questions. That means, in every reading
passage, there will be about one or two detail-locating questions. It is quite simple, however, to
solve this seemingly difficult question if you are able locate the detail tested. The right answer
choice is rewritten from certain sentence in the passage by changing some words or phrases. For
example, test writer will change some words from adjective to adverbial, from noun to gerund, or
just change to its synonym.
Strategy: How to locate
Below we will introduce the three-step method to locate detail.
(1) Before you locate the question to passage, you need to determine what to locate. Key words
are something that is mentioned both in the question and in passage. Then, what are key words?
Look at the following question:
Which of the following is mentioned in the passage as a disadvantage of storing artifacts
in museum basements?
Here, key words will not be any word or phrase of "which of the following is mentioned in the
passage as", but will be "disadvantage" from "disadvantage of storing artifacts in museum
basements".
We call this step as Defining Key Words.
(2) After you define key words, you are turning to the original passage. Sometimes, the key words
will appear several times in different parts of the passage. Where should you refer to? Generally,
you should locate the key words to the sentence in which key words first appear. After all, you
have only several minutes to complete a passage.
(3) When you determined which sentence (sometimes, two or more sentences) to locate, get
some idea, then quickly refer to the answer choices. Do not spend too much time analyzing this
sentence since it may be too long or complicated to understand. If the choice mentions something
that only appears in other part of the passage, eliminate it. Also eliminate the choice that just
repeats the words or phrases from original passage.
B. Listing
The other type of Recall Question is listing. As the name indicates, Listing Question requires you
to identify some people, actions, or situations that are enumerated in the passage.
36
Here are some Listing Questions:
z According to the passage, senior managers use intuition in all of the following ways
EXCEPT to:
z According to the passage, critics of the Ewha women's studies program cited the program
as a threat to which of the following?
I. National identity
II. National unification
III. Economic development
IV. Family integrity
(A) I only (B) I and II only (C) I, II, and III only
(D) II, III, and IV only (E) I, II, III, and IV
To solve this type of question, you should first name the key word from the stimulus, and locate it
to original passage. Then, you will find some lists that are similar to the answer choices. Carefully
compare those lists one by one to the answer choices. Use POE to eliminate incorrect choice,
until you find the right one.
37
3. Inference Questions
Inference question is the second most common. Unlike recall question, inference questions
require you to go beyond the passage. That means, the correct answer must say more than what
is said in the passage. Beware of same language traps with these questions: the correct answer
will often both paraphrase and extend a statement in the passage, but it will not directly quote it. If
you are puzzled how to determine whether a detail question is recall question or inference
question, pay attentions to the way the question asks. Generally, inference question will include
some word, such as infer, suggest and imply that indicates what kind of question it is.
z It can be inferred from the passage that...
z The passage/author suggests that.
z The passage/author implies that...
Since we must not directly refer to the original passage in answering inference question, we need
to decipher the inference. Next, we will show you how to reason from couples of sentence.
Technique 1 Reasoning by Word of Comparison
The question is asking about B, but you may be unable to directly identify the characters of B
even you have located B. Rather, the original sentence is discussing about A. Here, you should
turn to the word that indicates comparison between A and B. Some words that indicate strong
comparison are unlike, in contrast to, by contrast and compared with. When you can determine
the character of B, you can simultaneously determine A is B or non-B. Also, the passage may
compare two particular events by dates or places. The phrases could be "prior to 1975" or "since
mid-1970's".
Technique 2 Reasoning by Syllogism
In logics, Syllogism looks like this: every virtue is laudable; kindness is a virtue; therefore,
kindness is laudable. As we put it in more simple way, it may be "A→B and, then A→C". It may be
relatively easy to recognize A→B by locating the key word in the question, but it will always take
some time to identify B→C, since they may be located in other part of the place. So pay attention
to the pronouns (it or they) and the nouns with definite article "the" since they often serve as B.
The fact that reducing price can generate a competitive advantage for a company does not
mean that every reduction in price will create such an advantage. Price reduction, like
improvement in service, must be balanced against other types of efforts on the basis of direct,
tangible benefits such as increased revenues. If a company is already effectively on a par
with its competitors because it provides product at an acceptable price and keeps customers
from leaving at an unacceptable rate, then reduction in price may not be effective, since price
is not necessarily the deciding factor for any customer in any situation.
38
This truth was not apparent to managers of one operating system software vendor, which
failed to improve its competitive position despite its attempt to reduce price. The software
managers did not recognize the level of customer inertia that arises from the inconvenience
of switching operating system. Nor did they analyze their reduction in price to determine
whether it would attract new customers by producing a new standard of price that would
excite customers or by proving difficult for competitors to copy.
The passage suggests which of the following about price charged by an operating system software
vendor prior to its strategy in reducing its price?
(A) It enabled the vendor to retain customers at an acceptable rate
(B) It threatened to weaken the vendor’s competitive position with respect to other operating
system software vendor
(C) It had already been reduced after having caused damage to the vendor’s reputation in the
past.
(D) It was slightly low to that of the vendor’s competitors.
(E) It needed to be reduced to attain parity with the software provided by competing vendors.
Here, the question asks for the situation of the vendor before price reduction. Only A can be
inferred from the passage. The original passage stated that “If a company is already effectively
on ….. keeps customers from leaving at an unacceptable rate…” and “This truth was not
apparent to managers of one operating system software vendor…” That means the vendor was
able to retain customers at an acceptable rate.
39
4. Critical Reasoning Question
Even in reading comprehension, you will encounter some critical reasoning questions: analogy,
assumption, weaken, and strengthen. Here, the whole passage is an argument with premises,
assumptions and conclusions. The question asks you to identify the reasoning, critique the
argument or recognize the potential assumption. When you need to do is also to first locate the
conclusion to particular sentence of the passage, then identify the evidence and conclusion. The
premise (or evidence) could be near to the conclusion, or in other part of the 3-4 paragraph
passage. So, it is more difficult than its counterpart in critical reasoning section. That's why it
appears more often in high difficult level screen.
A. Analogy
Also known as application question, analogy question requires you to identify the author's
reasoning somewhere in the passage and then ask you to select one from the following five
answer choices that reasons as that is presented in the passage.
The answer choices are generally long and complicated, but they are not so difficult to
understand. After you locate the details to certain sentences in the passage, try to identify the
reasoning, and then turn to the answer choices. Fortunately, once you identify the reasoning, you
will quickly get the right answer since there are great differences among these five choices.
B. Assumption/Weaken/Strengthen
Weaken, Support, and Assumption are the other three types of question you are expected to
solve in critical reasoning question. As we said above, you need to evaluate the argument and
identify the assumptions. Typical questions would be:
z Which of the following, if true, would most weaken the theory proposed by Snyder et al?
z Which of the following, if true, would most strongly support Keyssar's findings as they are
described by the author?
Sample Question
The cutting-edge science is ringing alarm bells. Avian flu virus picked up by pigs can swap
genetic materials with another flu virus already in the pig and become a new, hitherto
unknown flu virus for which no person, no animal has preexisting immunity. The kind of virus
causes a pandemic because it spreads from human to human.
If you took a peek into history, it turns out that previous influenza pandemics have similar
scenarios. The greatest influenza pandemic in 1918 caused more than 20 million deaths of
soldiers stationed in France. The last influenza pandemic was in 1968, known as the Hong
Kong flu (H3N2). Thousands of deaths and millions were infected worldwide.
40
The other examples are the Nipah virus and Japanese Encephalitis virus, which find pigs to
be good hosts. With JE, the virus circulates in the blood of infected pigs. When infected pigs
are bitten by Culex mosquitoes, the virus replicates in the mosquito's gut. The next time the
mosquito bites a human, the virus is passed on. The pig doesn't get sick as such. The Nipah
virus causes pneumonia symptoms in pigs. In humans, it causes encephalitis, and humans
catch it only with direct contact with infected pigs. Symptoms range from mild headache to
permanent brain damage, and can be fatal.
It's merely a phenomenon of nature that the pig is the "mixing vessel" for the new germ. But
make no mistake, the pig is not the villain, neither is the chicken. It's actually us, and our
horrible farm practices, outdated agricultural policy and, most of all, reckless disregard of our
ecology and environment. "Hygiene and management can control what eventually happens,"
says Lam. "Good farming practice will prevent serious outbreaks and infection to humans."
Despite knowing that, animal diseases and the possibility of transmission to humans are
becoming quite alarming. Of the 35 new emerging diseases in the last 20 years, more than
70 per cent involved animals.
In fact, what we may have done is unwittingly create the perfect launch pad for an influenza
pandemic that will likely kill large numbers of people across the globe. Although scientists
say it's impossible to predict the odds that the virus will alter its genetic form radically enough
to start leaping from human to human, the longer H5N1 is out there killing chickens, the
higher the chances are.
All of the following situations are similar to the spread of avian flu virus described in the first
paragraph EXCEPT:
(A) The BT2 spread from a pig to another pig, and thus causes significant disease in pig.
(B) The AIDS viruses transferred from monkeys to man and spread across the world.
(C) The SARS virus originates from some wildlife and is picked up by civet cats from which
humans got it.
(D) Nipah virus circulates in the blood of infected pig, which is bitten by Culex mosquitoes, the
virus replicates in the mosquito's gut. The next time the mosquito bites a human, the virus
is passed on.
(E) H5N1 starts in chickens and leaps from human to human.
The question requires you to recognize a situation that is not similar to the spear of avian flu.
Before considering following answer choices, we fist define its rationale. It is something like this:
Avian flu virus picked up by pigs and is transferred to human. All of the situations described in the
answer choices are similar to it ex that in choice A (from animal to animal). Therefore, A is the
best answer.
41
5. Difficult-to-locate Question
Some question does not ask for the central idea of a passage. Rather, it requires you to draw a
conclusion based on the passage:
z According to the passage, which of the following is the author most likely to agree with?
z The passage supplies information that would answer which of the following questions?
Unlike Recall Question or Inference Question, Difficult-to-locate Question does not contain key
words that you can use to locate the details tested. In order to solve this type of question, you
have to skim through the passage again and again until you get the right answer. Eliminating
wrong choices often take considerable time since the answer choices are often too long and
complicated to understand. That is why most test takers regard difficult-to-locate question as the
most difficult one in reading comprehension. The good news is that if you encounter several
questions like these, then you probably get a high score since questions are presented based on
your performance on the previous questions.
Sample Question
Indian firms have achieved the highest levels of efficiency in the world software outsourcing
industry. Some researchers have assumed that Indian firms use the same programming
languages and techniques as Chinese firms but have benefited from their familiarity with
English, the language used to write software code. However, if this were true, then one
would expect software vendors in Hong Kong, where most people speak English, to perform
not worse than do Indian vendors. However, this is obviously not the case.
Other researchers link high Indian productivity to higher levels of human resource investment
per engineer. But a historical perspective leads to a different conclusion. When the two top
Indian vendors matched and then doubled Chinese productivity levels in the mid-eighties,
human resource investment per employee was comparable to that of Chinese vendors.
Furthermore, by the late eighties, the amount of fixed assets required to develop one
software package was roughly equivalent in India and in the China. Since human resource
investment was not higher in India, it had to be other factors that led to higher productivity.
A more fruitful explanation may lie with Indian strategic approach in outsourcing. Indian
software vendors did not simply seek outsourced contract more effectively: they made
aggressive strategic in outsourcing. For instance, most software firms of India were initially
set up to outsource the contract in western countries, such as United States. By contrary,
most Chinese firms seem to position their business in China, a promising yet
under-developed market. However, rampant piracy in China took almost 90 percents of
potential market, making it impossible for most Chinese firms to obtain sufficient
42
compensation for the investment on development and research, let alone thrive in
competitive environment.
According to the passage, which of the following statements is true of Indian software developers?
(A) Their productivity levels did not equal those of Chinese software engineers until the late
eighties.
(B) Their high efficiency levels are a direct result of English language familiarity.
(C) They develop component-specific software.
(D) They are built to outsource the western orders.
(E) They develop more packages of software than do those in Chinese developers.
In the middle of the last paragraph, the author states that “For instance, most software firms of
India were initially set up to outsource the contract in western countries, such as United
States.” Thus, the best answer is D.
43
Section 6: Six test points
While four-step procedure helps you to understand a passage and the five types of question
guide you how ETS test the understanding of the passage, the six test points will in advance
introduce what would be tested even before you read the questions. As you are reading the
passage, keep alarm to certain words or phrases since they would later act as clues for
answering the following questions. We call these signal words or phrases as test points.
In the following passage, we will introduce you the six most common test points in reading
comprehension. Once you become familiar with these test points, you will get advantage in speed
to come up with the right answer choice.
1. Comparison
Words or phrases: like, unlike, in contrast to, similarly
Question Type: recall question, inference question
Here is an example:
The fact that reducing price can generate a competitive advantage for a company does not
mean that every reduction in price will create such an advantage. Price reduction, like
improvement in service, must be balanced against other types of efforts on the basis of
direct, tangible benefits such as increased revenues. If a company is already effectively on a
par with its competitors because it provides product at an acceptable price and keeps
customers from leaving at an unacceptable rate, then reduction in price may not be effective,
since price is not necessarily the deciding factor for any customer in any situation.
This truth was not apparent to managers of one operating system software vendor, which
failed to improve its competitive position despite its attempt to reduce price. The software
managers did not recognize the level of customer inertia that arises from the inconvenience
of switching operating system. Nor did they analyze their reduction in price to determine
whether it would attract new customers by producing a new standard of price that would
excite customers or by proving difficult for competitors to copy.
Sample question
According to the passage, reduction in price are comparable to improvement in service in
terms of the
(A) tangibility of the benefits that they tend to confer
(B) increased revenues that they ultimately produce
44
(C) basis on which they need to be weighed
(D) insufficient analysis that managers devote to them
(E) degree of competitive advantage that they are likely to provide
To answer this question, first locate the question to the second sentence of the passage. "Price
reduction, like improvement in service, must be balanced against other types of efforts on the
basis of direct, tangible benefits such as increased revenues." In other words, they are
comparable based on which they need to be weighed. Therefore, C is the correct answer.
45
2. Example & Listing
Words or phrase: such as, as well as, for example, for instance
Question type: Listing, Exampling
Let's look at a sample question for the same passage.
The fact that reducing price can generate a competitive advantage for a company does not
mean that every reduction in price will create such an advantage. Price reduction, like
improvement in service, must be balanced against other types of efforts on the basis of direct,
tangible benefits such as increased revenues. If a company is already effectively on a par
with its competitors because it provides product at an acceptable price and keeps customers
from leaving at an unacceptable rate, then reduction in price may not be effective, since price
is not necessarily the deciding factor for any customer in any situation.
This truth was not apparent to managers of one operating system software vendor,
which failed to improve its competitive position despite its attempt to reduce price. The
software managers did not recognize the level of customer inertia that arises from the
inconvenience of switching operating system. Nor did they analyze their reduction in price to
determine whether it would attract new customers by producing a new standard of price that
would excite customers or by proving difficult for competitors to copy.
The discussion of the operating system software vendor last paragraph serves which of the
following functions within the passage as a whole?
(A) It describes an exceptional case in which reduction in price actually failed to produce a
competitive advantage.
(B) It illustrates the pitfalls of choosing to reduce price at a time when business strategy is
needed more urgently in another area.
(C) It demonstrates the kind of analysis that managers apply when they choose one kind of
business strategy over another
(D) It supports the argument that strategies in certain aspects are more advantageous than
strategies in other aspects.
(E) It provides an example of the point about reduction in price made in the first paragraph.
Clearly, the author intends to prove his position that reduction in price does not necessarily
generate competitive advantage. E is the correct.
46
3. People, Date & Place
Phrase: in the nineteenth-century, prior to mid-1970's, Snyder proposed that.
Question: inference question, main idea question
Indian firms have achieved the highest levels of efficiency in the world software outsourcing
industry. Some researchers have assumed that Indian firms use the same programming
languages and techniques as Chinese firms but have benefited from their familiarity with
English, the language used to write software code. However, if this were true, then one
would expect software vendors in Hong Kong, where most people speak English, to perform
not worse than do Indian vendors. However, this is obviously not the case.
Other researchers link high Indian productivity to higher levels of human resource investment
per engineer. But a historical perspective leads to a different conclusion. When the two top
Indian vendors matched and then doubled Chinese productivity levels in the mid-eighties,
human resource investment per employee was comparable to that of Chinese vendors.
Furthermore, by the late eighties, the amount of fixed assets required to develop one
software package was roughly equivalent in India and in the China. Since human resource
investment was not higher in India, it had to be other factors that led to higher productivity.
A more fruitful explanation may lie with Indian strategic approach in outsourcing. Indian
software vendors did not simply seek outsourced contract more effectively: they made
aggressive strategic in outsourcing. For instance, most software firms of India were initially
set up to outsource the contract in western countries, such as United States. By contrary,
most Chinese firms seem to position their business in China, a promising yet
under-developed market. However, rampant piracy in China took almost 90 percents of
potential market, making it impossible for most Chinese firms to obtain sufficient
compensation for the investment on development and research, let alone thrive in
competitive environment.
The author suggests that if the researchers of India mentioned in paragraph 1 were correct, which
of the following would be the case?
(A) The computer used in India software firms would be different from the computer used in
China firms.
(B) Indian engineers would be trained to do several different programming jobs.
(C) Familiarity with English language would not have an influence on the productivity levels of
engineers.
(D) The engineers in India-run firms would have lower productivity levels if they have a poor
command of English.
47
(E) The production levels of India-run firms located in the China would be equal to those of
firms run by China firms.
If the researchers are correct, then the familiarity with English determines the productivity of
engineers. That is, if the engineers in India-run firms have a poor command of English they would
have lower productivity levels, as stated in choice D.
GMAT Verbal Study Guide - GMAT CAT.com ( PDFDrive ).pdf
GMAT Verbal Study Guide - GMAT CAT.com ( PDFDrive ).pdf
GMAT Verbal Study Guide - GMAT CAT.com ( PDFDrive ).pdf
GMAT Verbal Study Guide - GMAT CAT.com ( PDFDrive ).pdf
GMAT Verbal Study Guide - GMAT CAT.com ( PDFDrive ).pdf
GMAT Verbal Study Guide - GMAT CAT.com ( PDFDrive ).pdf
GMAT Verbal Study Guide - GMAT CAT.com ( PDFDrive ).pdf
GMAT Verbal Study Guide - GMAT CAT.com ( PDFDrive ).pdf
GMAT Verbal Study Guide - GMAT CAT.com ( PDFDrive ).pdf
GMAT Verbal Study Guide - GMAT CAT.com ( PDFDrive ).pdf
GMAT Verbal Study Guide - GMAT CAT.com ( PDFDrive ).pdf
GMAT Verbal Study Guide - GMAT CAT.com ( PDFDrive ).pdf
GMAT Verbal Study Guide - GMAT CAT.com ( PDFDrive ).pdf
GMAT Verbal Study Guide - GMAT CAT.com ( PDFDrive ).pdf
GMAT Verbal Study Guide - GMAT CAT.com ( PDFDrive ).pdf
GMAT Verbal Study Guide - GMAT CAT.com ( PDFDrive ).pdf
GMAT Verbal Study Guide - GMAT CAT.com ( PDFDrive ).pdf
GMAT Verbal Study Guide - GMAT CAT.com ( PDFDrive ).pdf
GMAT Verbal Study Guide - GMAT CAT.com ( PDFDrive ).pdf
GMAT Verbal Study Guide - GMAT CAT.com ( PDFDrive ).pdf
GMAT Verbal Study Guide - GMAT CAT.com ( PDFDrive ).pdf
GMAT Verbal Study Guide - GMAT CAT.com ( PDFDrive ).pdf
GMAT Verbal Study Guide - GMAT CAT.com ( PDFDrive ).pdf
GMAT Verbal Study Guide - GMAT CAT.com ( PDFDrive ).pdf
GMAT Verbal Study Guide - GMAT CAT.com ( PDFDrive ).pdf
GMAT Verbal Study Guide - GMAT CAT.com ( PDFDrive ).pdf
GMAT Verbal Study Guide - GMAT CAT.com ( PDFDrive ).pdf
GMAT Verbal Study Guide - GMAT CAT.com ( PDFDrive ).pdf
GMAT Verbal Study Guide - GMAT CAT.com ( PDFDrive ).pdf
GMAT Verbal Study Guide - GMAT CAT.com ( PDFDrive ).pdf
GMAT Verbal Study Guide - GMAT CAT.com ( PDFDrive ).pdf
GMAT Verbal Study Guide - GMAT CAT.com ( PDFDrive ).pdf
GMAT Verbal Study Guide - GMAT CAT.com ( PDFDrive ).pdf
GMAT Verbal Study Guide - GMAT CAT.com ( PDFDrive ).pdf
GMAT Verbal Study Guide - GMAT CAT.com ( PDFDrive ).pdf
GMAT Verbal Study Guide - GMAT CAT.com ( PDFDrive ).pdf
GMAT Verbal Study Guide - GMAT CAT.com ( PDFDrive ).pdf
GMAT Verbal Study Guide - GMAT CAT.com ( PDFDrive ).pdf
GMAT Verbal Study Guide - GMAT CAT.com ( PDFDrive ).pdf
GMAT Verbal Study Guide - GMAT CAT.com ( PDFDrive ).pdf
GMAT Verbal Study Guide - GMAT CAT.com ( PDFDrive ).pdf
GMAT Verbal Study Guide - GMAT CAT.com ( PDFDrive ).pdf
GMAT Verbal Study Guide - GMAT CAT.com ( PDFDrive ).pdf
GMAT Verbal Study Guide - GMAT CAT.com ( PDFDrive ).pdf
GMAT Verbal Study Guide - GMAT CAT.com ( PDFDrive ).pdf
GMAT Verbal Study Guide - GMAT CAT.com ( PDFDrive ).pdf
GMAT Verbal Study Guide - GMAT CAT.com ( PDFDrive ).pdf
GMAT Verbal Study Guide - GMAT CAT.com ( PDFDrive ).pdf
GMAT Verbal Study Guide - GMAT CAT.com ( PDFDrive ).pdf
GMAT Verbal Study Guide - GMAT CAT.com ( PDFDrive ).pdf
GMAT Verbal Study Guide - GMAT CAT.com ( PDFDrive ).pdf
GMAT Verbal Study Guide - GMAT CAT.com ( PDFDrive ).pdf
GMAT Verbal Study Guide - GMAT CAT.com ( PDFDrive ).pdf
GMAT Verbal Study Guide - GMAT CAT.com ( PDFDrive ).pdf
GMAT Verbal Study Guide - GMAT CAT.com ( PDFDrive ).pdf
GMAT Verbal Study Guide - GMAT CAT.com ( PDFDrive ).pdf
GMAT Verbal Study Guide - GMAT CAT.com ( PDFDrive ).pdf
GMAT Verbal Study Guide - GMAT CAT.com ( PDFDrive ).pdf
GMAT Verbal Study Guide - GMAT CAT.com ( PDFDrive ).pdf
GMAT Verbal Study Guide - GMAT CAT.com ( PDFDrive ).pdf
GMAT Verbal Study Guide - GMAT CAT.com ( PDFDrive ).pdf
GMAT Verbal Study Guide - GMAT CAT.com ( PDFDrive ).pdf
GMAT Verbal Study Guide - GMAT CAT.com ( PDFDrive ).pdf
GMAT Verbal Study Guide - GMAT CAT.com ( PDFDrive ).pdf
GMAT Verbal Study Guide - GMAT CAT.com ( PDFDrive ).pdf
GMAT Verbal Study Guide - GMAT CAT.com ( PDFDrive ).pdf
GMAT Verbal Study Guide - GMAT CAT.com ( PDFDrive ).pdf

Más contenido relacionado

Similar a GMAT Verbal Study Guide - GMAT CAT.com ( PDFDrive ).pdf

Chuck moorebook2012 01_27
Chuck moorebook2012 01_27Chuck moorebook2012 01_27
Chuck moorebook2012 01_27
juergenuk
 
NLC23 - Grade 8 Consolidation English Student Workbook - Final.pdf
NLC23 - Grade 8 Consolidation English Student Workbook - Final.pdfNLC23 - Grade 8 Consolidation English Student Workbook - Final.pdf
NLC23 - Grade 8 Consolidation English Student Workbook - Final.pdf
lea sabroso
 
3016 all-2007-dist
3016 all-2007-dist3016 all-2007-dist
3016 all-2007-dist
NYversity
 
93613757 how-to-teach
93613757 how-to-teach93613757 how-to-teach
93613757 how-to-teach
Edgar Sanchez
 
A_Practical_Introduction_to_Python_Programming_Heinold.pdf
A_Practical_Introduction_to_Python_Programming_Heinold.pdfA_Practical_Introduction_to_Python_Programming_Heinold.pdf
A_Practical_Introduction_to_Python_Programming_Heinold.pdf
TariqSaeed80
 

Similar a GMAT Verbal Study Guide - GMAT CAT.com ( PDFDrive ).pdf (20)

Guide math k_3_nsn-2
Guide math k_3_nsn-2Guide math k_3_nsn-2
Guide math k_3_nsn-2
 
Chuck moorebook2012 01_27
Chuck moorebook2012 01_27Chuck moorebook2012 01_27
Chuck moorebook2012 01_27
 
Teach yourself logic 2017
Teach yourself logic 2017Teach yourself logic 2017
Teach yourself logic 2017
 
Teach yourself logic 2017
Teach yourself logic 2017Teach yourself logic 2017
Teach yourself logic 2017
 
Learn python the right way
Learn python the right wayLearn python the right way
Learn python the right way
 
Math for Smart Kids Gr.2
Math for Smart Kids Gr.2Math for Smart Kids Gr.2
Math for Smart Kids Gr.2
 
NP problems
NP problemsNP problems
NP problems
 
Nguyễn Nho Vĩnh
Nguyễn Nho VĩnhNguyễn Nho Vĩnh
Nguyễn Nho Vĩnh
 
NLC23 - Grade 8 Consolidation English Student Workbook - Final.pdf
NLC23 - Grade 8 Consolidation English Student Workbook - Final.pdfNLC23 - Grade 8 Consolidation English Student Workbook - Final.pdf
NLC23 - Grade 8 Consolidation English Student Workbook - Final.pdf
 
NLC23 - Grade 8 Consolidation English Student Workbook - Final.pdf
NLC23 - Grade 8 Consolidation English Student Workbook - Final.pdfNLC23 - Grade 8 Consolidation English Student Workbook - Final.pdf
NLC23 - Grade 8 Consolidation English Student Workbook - Final.pdf
 
3016 all-2007-dist
3016 all-2007-dist3016 all-2007-dist
3016 all-2007-dist
 
93613757 how-to-teach
93613757 how-to-teach93613757 how-to-teach
93613757 how-to-teach
 
Grammar1
Grammar1Grammar1
Grammar1
 
Passport workbook 2009 10 web version
Passport workbook 2009 10 web versionPassport workbook 2009 10 web version
Passport workbook 2009 10 web version
 
Georgia Common Core Coach, CCGPS Edition, Composition, Level I
Georgia Common Core Coach, CCGPS Edition, Composition, Level IGeorgia Common Core Coach, CCGPS Edition, Composition, Level I
Georgia Common Core Coach, CCGPS Edition, Composition, Level I
 
A_Practical_Introduction_to_Python_Programming_Heinold.pdf
A_Practical_Introduction_to_Python_Programming_Heinold.pdfA_Practical_Introduction_to_Python_Programming_Heinold.pdf
A_Practical_Introduction_to_Python_Programming_Heinold.pdf
 
A practical introduction_to_python_programming_heinold
A practical introduction_to_python_programming_heinoldA practical introduction_to_python_programming_heinold
A practical introduction_to_python_programming_heinold
 
A Practical Introduction To Python Programming
A Practical Introduction To Python ProgrammingA Practical Introduction To Python Programming
A Practical Introduction To Python Programming
 
A practical introduction_to_python_programming_heinold
A practical introduction_to_python_programming_heinoldA practical introduction_to_python_programming_heinold
A practical introduction_to_python_programming_heinold
 
A_Practical_Introduction_to_Python_Programming_Heinold.pdf
A_Practical_Introduction_to_Python_Programming_Heinold.pdfA_Practical_Introduction_to_Python_Programming_Heinold.pdf
A_Practical_Introduction_to_Python_Programming_Heinold.pdf
 

Más de Sumni Uchiha

chemistry .pdf
chemistry .pdfchemistry .pdf
chemistry .pdf
Sumni Uchiha
 
The_Official_Guide_for_GMAT_Quantitative_Review,_2nd_Edition_PDFDrive.pdf
The_Official_Guide_for_GMAT_Quantitative_Review,_2nd_Edition_PDFDrive.pdfThe_Official_Guide_for_GMAT_Quantitative_Review,_2nd_Edition_PDFDrive.pdf
The_Official_Guide_for_GMAT_Quantitative_Review,_2nd_Edition_PDFDrive.pdf
Sumni Uchiha
 
Advanced-Quant-Manhattan.pdf
Advanced-Quant-Manhattan.pdfAdvanced-Quant-Manhattan.pdf
Advanced-Quant-Manhattan.pdf
Sumni Uchiha
 
gmat-exam-success.pdf
gmat-exam-success.pdfgmat-exam-success.pdf
gmat-exam-success.pdf
Sumni Uchiha
 
thebrainbook.pdf
thebrainbook.pdfthebrainbook.pdf
thebrainbook.pdf
Sumni Uchiha
 
The Ultimate Price Action Trading Guide - Atanas Matov_010821220629.pdf
The Ultimate Price Action Trading Guide - Atanas Matov_010821220629.pdfThe Ultimate Price Action Trading Guide - Atanas Matov_010821220629.pdf
The Ultimate Price Action Trading Guide - Atanas Matov_010821220629.pdf
Sumni Uchiha
 
raghuram-rajan-penguin-press-2019_221120225833.pdf
raghuram-rajan-penguin-press-2019_221120225833.pdfraghuram-rajan-penguin-press-2019_221120225833.pdf
raghuram-rajan-penguin-press-2019_221120225833.pdf
Sumni Uchiha
 
philip_e._tetlock_-_superforecasting_the_art_and_science_of_prediction.pdf
philip_e._tetlock_-_superforecasting_the_art_and_science_of_prediction.pdfphilip_e._tetlock_-_superforecasting_the_art_and_science_of_prediction.pdf
philip_e._tetlock_-_superforecasting_the_art_and_science_of_prediction.pdf
Sumni Uchiha
 
Nutrition_and_Metabolism_in_Sports,_Exercise_and_HealthMss_Telegram_050221150...
Nutrition_and_Metabolism_in_Sports,_Exercise_and_HealthMss_Telegram_050221150...Nutrition_and_Metabolism_in_Sports,_Exercise_and_HealthMss_Telegram_050221150...
Nutrition_and_Metabolism_in_Sports,_Exercise_and_HealthMss_Telegram_050221150...
Sumni Uchiha
 
Naked - A Cultural History of American Nudism_140321221956.pdf
Naked - A Cultural History of American Nudism_140321221956.pdfNaked - A Cultural History of American Nudism_140321221956.pdf
Naked - A Cultural History of American Nudism_140321221956.pdf
Sumni Uchiha
 
misbehaving.pdf
misbehaving.pdfmisbehaving.pdf
misbehaving.pdf
Sumni Uchiha
 
Implanting_Strategic_Management_by_H_Igor_Ansoff,_Daniel_Kipley_160222190211.pdf
Implanting_Strategic_Management_by_H_Igor_Ansoff,_Daniel_Kipley_160222190211.pdfImplanting_Strategic_Management_by_H_Igor_Ansoff,_Daniel_Kipley_160222190211.pdf
Implanting_Strategic_Management_by_H_Igor_Ansoff,_Daniel_Kipley_160222190211.pdf
Sumni Uchiha
 
How to Make Money in Day Trading by Mandar Jamsandekar [Jamsandekar, Mandar] ...
How to Make Money in Day Trading by Mandar Jamsandekar [Jamsandekar, Mandar] ...How to Make Money in Day Trading by Mandar Jamsandekar [Jamsandekar, Mandar] ...
How to Make Money in Day Trading by Mandar Jamsandekar [Jamsandekar, Mandar] ...
Sumni Uchiha
 
history of ancient karnataka.pdf
history of ancient karnataka.pdfhistory of ancient karnataka.pdf
history of ancient karnataka.pdf
Sumni Uchiha
 
Handbook_of_Nutrition_in_the_Aged_Fourth_Edition_Mss_Telegram_wattsapp_220620...
Handbook_of_Nutrition_in_the_Aged_Fourth_Edition_Mss_Telegram_wattsapp_220620...Handbook_of_Nutrition_in_the_Aged_Fourth_Edition_Mss_Telegram_wattsapp_220620...
Handbook_of_Nutrition_in_the_Aged_Fourth_Edition_Mss_Telegram_wattsapp_220620...
Sumni Uchiha
 
Foster & McChesney - The Endless Crisis; How Monopoly-Finance Capital Produce...
Foster & McChesney - The Endless Crisis; How Monopoly-Finance Capital Produce...Foster & McChesney - The Endless Crisis; How Monopoly-Finance Capital Produce...
Foster & McChesney - The Endless Crisis; How Monopoly-Finance Capital Produce...
Sumni Uchiha
 
food_biochemistry_and_food_processing_Mss_Telegram_wattsapp_00201098772552_14...
food_biochemistry_and_food_processing_Mss_Telegram_wattsapp_00201098772552_14...food_biochemistry_and_food_processing_Mss_Telegram_wattsapp_00201098772552_14...
food_biochemistry_and_food_processing_Mss_Telegram_wattsapp_00201098772552_14...
Sumni Uchiha
 
Fennema's_food_chemistryMss_Telegram_wattsapp_00201098772552_140520120504.pdf
Fennema's_food_chemistryMss_Telegram_wattsapp_00201098772552_140520120504.pdfFennema's_food_chemistryMss_Telegram_wattsapp_00201098772552_140520120504.pdf
Fennema's_food_chemistryMss_Telegram_wattsapp_00201098772552_140520120504.pdf
Sumni Uchiha
 

Más de Sumni Uchiha (20)

chemistry .pdf
chemistry .pdfchemistry .pdf
chemistry .pdf
 
The_Official_Guide_for_GMAT_Quantitative_Review,_2nd_Edition_PDFDrive.pdf
The_Official_Guide_for_GMAT_Quantitative_Review,_2nd_Edition_PDFDrive.pdfThe_Official_Guide_for_GMAT_Quantitative_Review,_2nd_Edition_PDFDrive.pdf
The_Official_Guide_for_GMAT_Quantitative_Review,_2nd_Edition_PDFDrive.pdf
 
Advanced-Quant-Manhattan.pdf
Advanced-Quant-Manhattan.pdfAdvanced-Quant-Manhattan.pdf
Advanced-Quant-Manhattan.pdf
 
gmat-exam-success.pdf
gmat-exam-success.pdfgmat-exam-success.pdf
gmat-exam-success.pdf
 
trading habits.pdf
trading habits.pdftrading habits.pdf
trading habits.pdf
 
thebrainbook.pdf
thebrainbook.pdfthebrainbook.pdf
thebrainbook.pdf
 
The Ultimate Price Action Trading Guide - Atanas Matov_010821220629.pdf
The Ultimate Price Action Trading Guide - Atanas Matov_010821220629.pdfThe Ultimate Price Action Trading Guide - Atanas Matov_010821220629.pdf
The Ultimate Price Action Trading Guide - Atanas Matov_010821220629.pdf
 
raghuram-rajan-penguin-press-2019_221120225833.pdf
raghuram-rajan-penguin-press-2019_221120225833.pdfraghuram-rajan-penguin-press-2019_221120225833.pdf
raghuram-rajan-penguin-press-2019_221120225833.pdf
 
philip_e._tetlock_-_superforecasting_the_art_and_science_of_prediction.pdf
philip_e._tetlock_-_superforecasting_the_art_and_science_of_prediction.pdfphilip_e._tetlock_-_superforecasting_the_art_and_science_of_prediction.pdf
philip_e._tetlock_-_superforecasting_the_art_and_science_of_prediction.pdf
 
Nutrition_and_Metabolism_in_Sports,_Exercise_and_HealthMss_Telegram_050221150...
Nutrition_and_Metabolism_in_Sports,_Exercise_and_HealthMss_Telegram_050221150...Nutrition_and_Metabolism_in_Sports,_Exercise_and_HealthMss_Telegram_050221150...
Nutrition_and_Metabolism_in_Sports,_Exercise_and_HealthMss_Telegram_050221150...
 
Naked - A Cultural History of American Nudism_140321221956.pdf
Naked - A Cultural History of American Nudism_140321221956.pdfNaked - A Cultural History of American Nudism_140321221956.pdf
Naked - A Cultural History of American Nudism_140321221956.pdf
 
misbehaving.pdf
misbehaving.pdfmisbehaving.pdf
misbehaving.pdf
 
Implanting_Strategic_Management_by_H_Igor_Ansoff,_Daniel_Kipley_160222190211.pdf
Implanting_Strategic_Management_by_H_Igor_Ansoff,_Daniel_Kipley_160222190211.pdfImplanting_Strategic_Management_by_H_Igor_Ansoff,_Daniel_Kipley_160222190211.pdf
Implanting_Strategic_Management_by_H_Igor_Ansoff,_Daniel_Kipley_160222190211.pdf
 
How to Make Money in Day Trading by Mandar Jamsandekar [Jamsandekar, Mandar] ...
How to Make Money in Day Trading by Mandar Jamsandekar [Jamsandekar, Mandar] ...How to Make Money in Day Trading by Mandar Jamsandekar [Jamsandekar, Mandar] ...
How to Make Money in Day Trading by Mandar Jamsandekar [Jamsandekar, Mandar] ...
 
history of ancient karnataka.pdf
history of ancient karnataka.pdfhistory of ancient karnataka.pdf
history of ancient karnataka.pdf
 
Handbook_of_Nutrition_in_the_Aged_Fourth_Edition_Mss_Telegram_wattsapp_220620...
Handbook_of_Nutrition_in_the_Aged_Fourth_Edition_Mss_Telegram_wattsapp_220620...Handbook_of_Nutrition_in_the_Aged_Fourth_Edition_Mss_Telegram_wattsapp_220620...
Handbook_of_Nutrition_in_the_Aged_Fourth_Edition_Mss_Telegram_wattsapp_220620...
 
Foster & McChesney - The Endless Crisis; How Monopoly-Finance Capital Produce...
Foster & McChesney - The Endless Crisis; How Monopoly-Finance Capital Produce...Foster & McChesney - The Endless Crisis; How Monopoly-Finance Capital Produce...
Foster & McChesney - The Endless Crisis; How Monopoly-Finance Capital Produce...
 
food_biochemistry_and_food_processing_Mss_Telegram_wattsapp_00201098772552_14...
food_biochemistry_and_food_processing_Mss_Telegram_wattsapp_00201098772552_14...food_biochemistry_and_food_processing_Mss_Telegram_wattsapp_00201098772552_14...
food_biochemistry_and_food_processing_Mss_Telegram_wattsapp_00201098772552_14...
 
Fennema's_food_chemistryMss_Telegram_wattsapp_00201098772552_140520120504.pdf
Fennema's_food_chemistryMss_Telegram_wattsapp_00201098772552_140520120504.pdfFennema's_food_chemistryMss_Telegram_wattsapp_00201098772552_140520120504.pdf
Fennema's_food_chemistryMss_Telegram_wattsapp_00201098772552_140520120504.pdf
 
Deep_Nutrition_Why_Your_Genes_Need_Traditional_Food_2017_Mss_Telegram_2603211...
Deep_Nutrition_Why_Your_Genes_Need_Traditional_Food_2017_Mss_Telegram_2603211...Deep_Nutrition_Why_Your_Genes_Need_Traditional_Food_2017_Mss_Telegram_2603211...
Deep_Nutrition_Why_Your_Genes_Need_Traditional_Food_2017_Mss_Telegram_2603211...
 

Último

Seal of Good Local Governance (SGLG) 2024Final.pptx
Seal of Good Local Governance (SGLG) 2024Final.pptxSeal of Good Local Governance (SGLG) 2024Final.pptx
Seal of Good Local Governance (SGLG) 2024Final.pptx
negromaestrong
 
Gardella_Mateo_IntellectualProperty.pdf.
Gardella_Mateo_IntellectualProperty.pdf.Gardella_Mateo_IntellectualProperty.pdf.
Gardella_Mateo_IntellectualProperty.pdf.
MateoGardella
 
Beyond the EU: DORA and NIS 2 Directive's Global Impact
Beyond the EU: DORA and NIS 2 Directive's Global ImpactBeyond the EU: DORA and NIS 2 Directive's Global Impact
Beyond the EU: DORA and NIS 2 Directive's Global Impact
PECB
 
An Overview of Mutual Funds Bcom Project.pdf
An Overview of Mutual Funds Bcom Project.pdfAn Overview of Mutual Funds Bcom Project.pdf
An Overview of Mutual Funds Bcom Project.pdf
SanaAli374401
 

Último (20)

Mehran University Newsletter Vol-X, Issue-I, 2024
Mehran University Newsletter Vol-X, Issue-I, 2024Mehran University Newsletter Vol-X, Issue-I, 2024
Mehran University Newsletter Vol-X, Issue-I, 2024
 
SOCIAL AND HISTORICAL CONTEXT - LFTVD.pptx
SOCIAL AND HISTORICAL CONTEXT - LFTVD.pptxSOCIAL AND HISTORICAL CONTEXT - LFTVD.pptx
SOCIAL AND HISTORICAL CONTEXT - LFTVD.pptx
 
Seal of Good Local Governance (SGLG) 2024Final.pptx
Seal of Good Local Governance (SGLG) 2024Final.pptxSeal of Good Local Governance (SGLG) 2024Final.pptx
Seal of Good Local Governance (SGLG) 2024Final.pptx
 
Key note speaker Neum_Admir Softic_ENG.pdf
Key note speaker Neum_Admir Softic_ENG.pdfKey note speaker Neum_Admir Softic_ENG.pdf
Key note speaker Neum_Admir Softic_ENG.pdf
 
APM Welcome, APM North West Network Conference, Synergies Across Sectors
APM Welcome, APM North West Network Conference, Synergies Across SectorsAPM Welcome, APM North West Network Conference, Synergies Across Sectors
APM Welcome, APM North West Network Conference, Synergies Across Sectors
 
fourth grading exam for kindergarten in writing
fourth grading exam for kindergarten in writingfourth grading exam for kindergarten in writing
fourth grading exam for kindergarten in writing
 
ICT Role in 21st Century Education & its Challenges.pptx
ICT Role in 21st Century Education & its Challenges.pptxICT Role in 21st Century Education & its Challenges.pptx
ICT Role in 21st Century Education & its Challenges.pptx
 
Advanced Views - Calendar View in Odoo 17
Advanced Views - Calendar View in Odoo 17Advanced Views - Calendar View in Odoo 17
Advanced Views - Calendar View in Odoo 17
 
Presentation by Andreas Schleicher Tackling the School Absenteeism Crisis 30 ...
Presentation by Andreas Schleicher Tackling the School Absenteeism Crisis 30 ...Presentation by Andreas Schleicher Tackling the School Absenteeism Crisis 30 ...
Presentation by Andreas Schleicher Tackling the School Absenteeism Crisis 30 ...
 
This PowerPoint helps students to consider the concept of infinity.
This PowerPoint helps students to consider the concept of infinity.This PowerPoint helps students to consider the concept of infinity.
This PowerPoint helps students to consider the concept of infinity.
 
microwave assisted reaction. General introduction
microwave assisted reaction. General introductionmicrowave assisted reaction. General introduction
microwave assisted reaction. General introduction
 
Gardella_Mateo_IntellectualProperty.pdf.
Gardella_Mateo_IntellectualProperty.pdf.Gardella_Mateo_IntellectualProperty.pdf.
Gardella_Mateo_IntellectualProperty.pdf.
 
Class 11th Physics NEET formula sheet pdf
Class 11th Physics NEET formula sheet pdfClass 11th Physics NEET formula sheet pdf
Class 11th Physics NEET formula sheet pdf
 
Beyond the EU: DORA and NIS 2 Directive's Global Impact
Beyond the EU: DORA and NIS 2 Directive's Global ImpactBeyond the EU: DORA and NIS 2 Directive's Global Impact
Beyond the EU: DORA and NIS 2 Directive's Global Impact
 
Basic Civil Engineering first year Notes- Chapter 4 Building.pptx
Basic Civil Engineering first year Notes- Chapter 4 Building.pptxBasic Civil Engineering first year Notes- Chapter 4 Building.pptx
Basic Civil Engineering first year Notes- Chapter 4 Building.pptx
 
Mattingly "AI & Prompt Design: Structured Data, Assistants, & RAG"
Mattingly "AI & Prompt Design: Structured Data, Assistants, & RAG"Mattingly "AI & Prompt Design: Structured Data, Assistants, & RAG"
Mattingly "AI & Prompt Design: Structured Data, Assistants, & RAG"
 
Grant Readiness 101 TechSoup and Remy Consulting
Grant Readiness 101 TechSoup and Remy ConsultingGrant Readiness 101 TechSoup and Remy Consulting
Grant Readiness 101 TechSoup and Remy Consulting
 
An Overview of Mutual Funds Bcom Project.pdf
An Overview of Mutual Funds Bcom Project.pdfAn Overview of Mutual Funds Bcom Project.pdf
An Overview of Mutual Funds Bcom Project.pdf
 
Measures of Central Tendency: Mean, Median and Mode
Measures of Central Tendency: Mean, Median and ModeMeasures of Central Tendency: Mean, Median and Mode
Measures of Central Tendency: Mean, Median and Mode
 
Paris 2024 Olympic Geographies - an activity
Paris 2024 Olympic Geographies - an activityParis 2024 Olympic Geographies - an activity
Paris 2024 Olympic Geographies - an activity
 

GMAT Verbal Study Guide - GMAT CAT.com ( PDFDrive ).pdf

  • 1. 1 GMAT Verbal Study Guide Table of Contents Chapter 1 Reading Comprehension ...............................................................................4 Section 1: One Principle .............................................................................................. 5 Section 2: Two Styles .................................................................................................. 8 1. Presentation....................................................................................................... 8 2. Argumentation .................................................................................................. 9 3. Organizational Structure ..................................................................................11 Section 3: Three Subjects........................................................................................... 14 1. Natural Science............................................................................................... 14 2. Social Science ................................................................................................. 15 3. Business Subject ............................................................................................. 17 Section 4: Four-step Process of Reading ................................................................... 20 1. Analyze the first paragraph............................................................................. 20 2. Skim the passage and get the author's main point........................................... 21 3. Diagram the organization of the passage........................................................ 23 4. Tackle the questions and correspondently refer to the passage. ..................... 26 Section 5: Five Types of Questions ........................................................................... 29 1. Main Idea Question......................................................................................... 30 2. Recall Question............................................................................................... 35 3. Inference Questions ........................................................................................ 37 4. Critical Reasoning Question ........................................................................... 39 5. Difficult-to-locate Question............................................................................ 41 Section 6: Six test points............................................................................................ 43 1. Comparison..................................................................................................... 43 2. Example & Listing.......................................................................................... 43 3. People, Date & Place ...................................................................................... 46 4. Words of Attitude and Transition ................................................................... 47 5. Counter-evidence Indicators ........................................................................... 49 6. Special Punctuation......................................................................................... 51 Chapter 2 Sentence Correction ....................................................................................53 Introduction................................................................................................................ 53
  • 2. 2 Three-step method ..................................................................................................... 54 Section 1: Subject-Verb Agreement .......................................................................... 56 Section 2: Verb Time Sequences ............................................................................... 57 Section 3: Modification.............................................................................................. 58 A. Adjective or adverb as a modifier.................................................................. 58 B. Clause as a modifier ....................................................................................... 58 C. A long phrase as a modifier............................................................................ 59 D. Appositive as a modifier ................................................................................ 60 Section 4: Parallelism ................................................................................................ 61 Section 5: Pronoun..................................................................................................... 62 Section 6: Comparisons ............................................................................................. 63 1. Quality Comparison........................................................................................ 63 2. Quantity Comparison...................................................................................... 64 3. Analogy........................................................................................................... 64 Section 7: Choice of Word......................................................................................... 66 Section 8: Idioms ....................................................................................................... 67 Section 9: Sentence Structure .................................................................................... 72 Section 10: Subjunctive Mood................................................................................... 74 Section 11: Ambiguity ............................................................................................... 75 Section 12: Redundancy ............................................................................................ 76 Section 13: Awkward................................................................................................. 77 Section 14: Logicality................................................................................................ 78 Chapter 3 Critical Reasoning.......................................................................................80 Section 1: Introduction to Critical Reasoning............................................................ 81 1. One Definition: Argument .............................................................................. 81 2. Four elements of an argument......................................................................... 82 3. Seven Common Fallacies................................................................................ 84 4. Three-element Rule......................................................................................... 86 5. Two Traps ....................................................................................................... 87 6. Five Answer Choices ...................................................................................... 88 Section 2: Six Types of Argument............................................................................. 90 1. Deductive Argument....................................................................................... 90 2. Generalization................................................................................................. 94 3. Analogy........................................................................................................... 95 4. Causal Reasoning............................................................................................ 96 5. Finding Assumption........................................................................................ 97 6. Business Thinking........................................................................................... 99 Section 3: Eight Types of Question ......................................................................... 100 1. Inference Question........................................................................................ 100 2. Assumption Question.................................................................................... 102 3. Strengthen Question...................................................................................... 104
  • 3. 3 4. Weaken Question.......................................................................................... 106 5. Paradox Question.......................................................................................... 108 6. Reasoning Question ...................................................................................... 109 7. Complete Question.........................................................................................110 8. Boldface Question..........................................................................................112
  • 4. 4 Chapter 1 Reading Comprehension Reading Comprehension on the test day On the test day, you will expect to see three or four reading passages, each followed with three or four questions. The passages presented depend on how well you are performing on the test. However, the questions presented for the same passage do not depend on your performance. In other word, after you are assigned a reading passage, the next question presented for the same passage will not base on your performance on the previous question. Why Reading Comprehension is a nightmare to most students? Most people find the reading compression difficult to prepare because the subject matter is unfamiliar and could be anything. In order to make sure that nobody can take advantages on a particular subject, the test-maker takes every effort to diversify the subjects of the three or four passages on your test day. As a result, obscure subject matter is chosen so that you will be tested, not on your knowledge of a particular subject, but the test-taking skills. In addition, the reading passage is not created like the one we see on magazine, newspaper, or textbook. Rather, it uses a highly compressed style. Subjects of the passages are generally excerpted from academic articles that were published tens of years ago. Usually the chosen article is heavily edited until it is cut down to about 300 words, about one-third its original length. Though it is difficult to read, the reading techniques introduced in the following passage will help you pick up the right answer even without understanding the reading passage. Chapter Preview In order to make it easier for you to prepare for GMAT, we have developed an interesting course for Reading Comprehension. You will find this chapter all in number, as the section number suggests. We hope this would help you learn the test prep strategies. Section 1: One Principle Section 2: Two Writing Styles Section 3: Three Subjects Section 4: Four-step Procedure for Attacking a Passage Section 5: Five Types of Question Section 6: Six Test Points
  • 5. 5 Section 1: One Principle Directions: The questions in this group are based on the content of a passage. After reading the passage, choose the best answer to each question. Answer all questions following the passage on the basis of what is stated or implied in the passage On the test day, you will see the above direction on computer screen. Most students disregard this instruction since it appears in every test. However, it introduces a basic principle you should follow in answering a reading comprehension question. When answering questions, you must refer each of them to some place in the passage. Don't rely on memory, since too many traps are used with these questions. Also, don't base on your daily life experiences or college knowledge. Remember, GMAT doesn't test any specific knowledge on business or other functions. Image if a question is based on some specific knowledge, then those with broad knowledge can take advantages. This definitely violates the rule of GMAT. The test-maker often fools you by creating stuff choices that contain reasonable statement based on basic knowledge or your life experience, not on the passage. If you find an answer choice contains the widely known reasoning or statement on the test day, eliminate those choices with hesitation. Let's look at a sample passage that discusses why the Indian software vendors perform better than their counterparts in China. Indian firms have achieved the highest levels of efficiency in the world software outsourcing industry. Some researchers have assumed that Indian firms use the same programming languages and techniques as Chinese firms but have benefited from their familiarity with English, the language used to write software code. However, if this were true, then one would expect software vendors in Hong Kong, where most people speak English, to perform not worse than do Indian vendors. However, this is obviously not the case. Other researchers link high Indian productivity to higher levels of human resource investment per engineer. But a historical perspective leads to a different conclusion. When the two top Indian vendors matched and then doubled Chinese productivity levels in the mid-eighties, human resource investment per employee was comparable to that of Chinese vendors. Furthermore, by the late eighties, the amount of fixed assets required to develop one software package was roughly equivalent in India and in the China. Since human resource investment was not higher in India, it had to be other factors that led to higher productivity. A more fruitful explanation may lie with Indian strategic approach in outsourcing. Indian software vendors did not simply seek outsourced contract more effectively: they made aggressive strategic in outsourcing. For instance, most software firms of India were initially set up to outsource the contract in western countries, such as United States. By contrary,
  • 6. 6 most Chinese firms seem to position their business in China, a promising yet under-developed market. However, rampant piracy in China took almost 90 percents of potential market, making it impossible for most Chinese firms to obtain sufficient compensation for the investment on development and research, let alone thrive in competitive environment. Now, let's look at a sample question: Which of the following statements concerning the productivity levels of engineers can be inferred from the passage? (A) Prior to the 1980’s, the productivity levels of the top Indian software firms were exceeded by those of Chinese software firms. (B) The official language of a country has a large effect on the productivity levels of its software developers. (C) During the late 1980’s and early 1990’s, productivity levels were comparable in China and India. (D) The greater the number of engineers that a software firm has, the higher a firm’s productivity level. (E) The amount of human resource investment made by software developers in their firms determines the level of productivity. If you do not refer to the original passage, you may pick up B. For test-takers who have some backgrounds in computer, it is obvious that being familiar with English will gain some advantage in writing program code. However, the correct answer is C. In conclusion, the directions can run out of your eyes on the test day, but should be rooted deeply in your heart at the beginning of your test preparation. Trap: Some choices just repeat the same words or phrases that you read in the passage. Keep alarm to these choices since in most cases, they are incorrect. Here is an example: The fact that reducing price can generate a competitive advantage for a company does not mean that every reduction in price will create such an advantage. Price reduction, like improvement in service, must be balanced against other types of efforts on the basis of direct, tangible benefits such as increased revenues. If a company is already effectively on a par with its competitors because it provides product at an acceptable price and keeps customers
  • 7. 7 from leaving at an unacceptable rate, then reduction in price may not be effective, since price is not necessarily the deciding factor for any customer in any situation. This truth was not apparent to managers of one operating system software vendor, which failed to improve its competitive position despite its attempt to reduce price. The software managers did not recognize the level of customer inertia that arises from the inconvenience of switching operating system. Nor did they analyze their reduction in price to determine whether it would attract new customers by producing a new standard of price that would excite customers or by proving difficult for competitors to copy. Sample question According to the passage, reduction in price are comparable to improvement in service in terms of the (A) tangibility of the benefits that they tend to confer (B) increased revenues that they ultimately produce (C) basis on which they need to be weighed (D) insufficient analysis that managers devote to them (E) degree of competitive advantage that they are likely to provide To answer this question, first locate the question to the second sentence of the passage. "Price reduction, like improvement in service, must be balanced against other types of efforts on the basis of direct, tangible benefits such as increased revenues." Now, go back to answer choices. Choice D and E are irrelevant to the original sentence, so eliminate them. Then, look at the choice A and B, both of them repeat the original sentences. (A) tangibility of the benefits that they tend to confer (B) increased revenues that they ultimately produce (C) basis on which they need to be weighed Both A and B seem to be correct. However, reduction in price is comparable to that of improvement in service in term of the basis on direct and tangible benefits, not on the tangibility or specific benefits of increased revenues. So neither A nor B is correct. Choice B does not repeat the same words, but address the basis for comparison. Therefore, C is the correct answer.
  • 8. 8 Section 2: Two Styles There is an endless number of writing techniques that authors use to present their ideas. However, there are only two writing styles used in a GMAT reading passage: presentation and argumentation. 1. Presentation This technique is to present an idea that the author will agree or at least partially agree. The author strengthens his position by citing relevant evidences, each related to other in a highly structured manner. We call this style of writing as presentation. Sometimes, the author sometime may intentionally contrast his position with an opposing view. But most often the author is just anticipating an objection, he will soon refute it. Here is a sample passage in presentation. China as a nation faces two major financial problems. First, eighty-four percent of state-owned enterprises do not generate profit. Government failed to collect money from such business. Rather, it has to appropriate substantial funds to these enterprises in order to prevent them from going bankrupt and thus resulting in high unemployment rate. Second, 203 million of civilians in countryside will not be able to gain pension after they retire due to the limited budget of government. I would like to make an outrageous suggestion that would at one stroke generate finance earnings and provide funds for civilians’ retirement. I would propose that government sells its holdings in state-owned enterprises on the open market. Such sales would provide substantial funds for village civilian’s pension. At the same, they could cut down financial burden on these state-owned enterprises. You might object that government would be deprived of the opportunity to share its enterprise’s profit if someday they make money. I agree. Sell holdings of enterprises that would never generate profit. But, you might reply, every enterprise that competes on the market has potential. Here we part company. Theoretically, you may be correct in claiming that every enterprise has the potential to make money. Practically, you are wrong. I refer to the thousands of state-owned enterprises that are not likely to make money. These companies are 100 percent held by the nation as a whole. Government officials are appointed as the chairman, CEO and president. The management was not responsible for the public interest, but for the nation as a whole. If there is no significant loss in business, they will soon be promoted back to the higher level position in government. If their companies perform great, these executives receive direct money compensation. However,
  • 9. 9 their salary, when combined with such compensation, will be far below that of their counterpart in private company. It would be unrealistic to suggest that village civilians would have sufficient funds if government’s shares were sold on the open market. But the demand for compensating the state-own enterprises would be substantially reduced. The author developed the above passage by first pointing out a problem, suggesting a solution, anticipating counter-position, illustrating an example, refuting a second solution, and further anticipating possible objections. Obviously, this writing technique is presentation. 2. Argumentation The second writing style is argumentation. This technique has a number of variations, but the most common and direct is to develop two to three ideas and then point out why one is better than the other or just simply refute all of them and developed the author's own idea. Some common tip-off sentences to this method of analysis are: z It was traditionally assumed... z It was once believed... z It was frequently assumed .. z It was universally accepted.. z Many scientists have argued... The passage that discusses Indian and Chinese software firms represents a typical argumentation. At the beginning, the author presented a phenomenon and gave an explanation, but refuted that explanation immediately. Indian firms have achieved the highest levels of efficiency in the world software outsourcing industry. Some researchers have assumed that Indian firms use the same programming languages and techniques as Chinese firms but have benefited from their familiarity with English, the language used to write software code. However, if this were true, then one would expect software vendors in Hong Kong, where most people speak English, to perform not worse than do Indian vendors. However, this is obviously not the case. Then, the second explanation was introduced, but was denied again in the same paragraph. Other researchers link high Indian productivity to higher levels of human resource investment per engineer. But a historical perspective leads to a different conclusion. When the two top Indian vendors matched and then doubled Chinese productivity levels in the mid-eighties, human resource investment per employee was comparable to that of Chinese vendors.
  • 10. 10 Furthermore, by the late eighties, the amount of fixed assets required to develop one software package was roughly equivalent in India and in the China. Since human resource investment was not higher in India, it had to be other factors that led to higher productivity. Finally, a more fruitful one is presented. The author used the remaining passage try to argue that this explanation is the correct one. A more fruitful explanation may lie with Indian strategic approach in outsourcing. Indian software vendors did not simply seek outsourced contract more effectively: they made aggressive strategic in outsourcing. For instance, most software firms of India were initially set up to outsource the contract in western countries, such as United States. By contrary, most Chinese firms seem to position their business in China, a promising yet under-developed market. However, rampant piracy in China took almost 90 percents of potential market, making it impossible for most Chinese firms to obtain sufficient compensation for the investment on development and research, let alone thrive in competitive environment. Why bother to identify the writing style? Be familiar with the author's writing techniques can help you diagram the mental road map of a passage, identify the author's intention to cite an evidence, main idea of a passage, and most importantly, pick up the right choice quickly and decisively. Let’s go back the passage that talks about whether price reduction can generate a competitive advantage. The fact that reducing price can generate a competitive advantage for a company does not mean that every reduction in price will create such an advantage. Price reduction, like improvement in service, must be balanced against other types of efforts on the basis of direct, tangible benefits such as increased revenues. If a company is already effectively on a par with its competitors because it provides product at an acceptable price and keeps customers from leaving at an unacceptable rate, then reduction in price may not be effective, since price is not necessarily the deciding factor for any customer in any situation. This truth was not apparent to managers of one operating system software vendor, which failed to improve its competitive position despite its attempt to reduce price. The software managers did not recognize the level of customer inertia that arises from the inconvenience of switching operating system. Nor did they analyze their reduction in price to determine whether it would attract new customers by producing a new standard of price that would excite customers or by proving difficult for competitors to copy. In the above passage, the author did not try to present his own position (presentation). If any, the position is that he does not agree with the fact that reduction in price can generate competitive
  • 11. 11 advantage for a company. In fact, the speaker here argued against a popular point of view by reasoning and examples (argumentation). Let's look at a sample question to see how to pick up a right choice on the basis of writing styles. The primary purpose of the passage is to (A) contrast possible outcomes of a type of business strategy (B) suggest more careful evaluation of a type of business strategy (C) illustrate various ways in which a type of business strategy could fail to enhance revenues (D) trace the general problems of a company to a certain type of business strategy (E) criticize the way in which managers tend to analyze the costs and benefits of business strategies This question asks you to summarize the passage's central idea. Which of the five choices is correct? Based on the verbs initiating the five choices, you can eliminate three of them: (A) incorrect. To contrast is to compare several things, but not to agree or disagree. (C) incorrect. To illustrate is to give example, not to agree or disagree. (D) incorrect. To trace is to track, not to agree or disagree. Choice E began with argumental word criticize, but isn't the correct choice because it addresses the detail. Therefore, B is the right answer: to argue that superior service does not generate competitive advantage is to suggest more careful evaluation of a type of business strategy (price reduction). 3. Organizational Structure There are two major patterns that the test-maker uses to reach a conclusion: general-to-specific and specific-to-general. Become familiar with these writing patterns can help you identify the main idea of a passage. A. General-to-Specific Structure This structure is widely used in GMAT reading passage. The test-writer first makes a general argument, and then supports it using a series of specific examples or reasoning, and finally summaries by reclaiming his general argument. Here is the structure: z General claim, followed by z first evidence or reasoning z second evidence or reasoning
  • 12. 12 z more evidence or reasoning Let's look at a passage of this structure: The fact that reducing price can generate a competitive advantage for a company does not mean that every reduction in price will create such an advantage. Price reduction, like improvement in service, must be balanced against other types of efforts on the basis of direct, tangible benefits such as increased revenues. If a company is already effectively on a par with its competitors because it provides product at an acceptable price and keeps customers from leaving at an unacceptable rate, then reduction in price may not be effective, since price is not necessarily the deciding factor for any customer in any situation. This truth was not apparent to managers of one operating system software vendor, which failed to improve its competitive position despite its attempt to reduce price. The software managers did not recognize the level of customer inertia that arises from the inconvenience of switching operating system. Nor did they analyze their reduction in price to determine whether it would attract new customers by producing a new standard of price that would excite customers or by proving difficult for competitors to copy. Here, the author presents his opinion at the beginning of the passage: reduction in price does not necessarily generate a competitive advantage. To support his idea, the author first made reasoning by comparing service improvement and price reduction. Then, in the second paragraph, the author used an example within operating system software industry to further address that reducing price did not improve competitive position. B. Specific-to-General Structure Contrast to the general-to-specific structure, the specific-to-general first presents a group of examples or reasoning and finally draw a conclusion. Here is the structure: z first example or reasoning z second example or reasoning z more example or reasoning z Conclusion The passage that discusses Indian software vendors was written in argumentation, and represents a typical passage in specific-to-general structure. Indian firms have achieved the highest levels of efficiency in the world software outsourcing industry. Some researchers have assumed that Indian firms use the same programming languages and techniques as Chinese firms but have benefited from their familiarity with
  • 13. 13 English, the language used to write software code. However, if this were true, then one would expect software vendors in Hong Kong, where most people speak English, to perform not worse than do Indian vendors. However, this is obviously not the case. Other researchers link high Indian productivity to higher levels of human resource investment per engineer. But a historical perspective leads to a different conclusion. When the two top Indian vendors matched and then doubled Chinese productivity levels in the mid-eighties, human resource investment per employee was comparable to that of Chinese vendors. Furthermore, by the late eighties, the amount of fixed assets required to develop one software package was roughly equivalent in India and in the China. Since human resource investment was not higher in India, it had to be other factors that led to higher productivity. A more fruitful explanation may lie with Indian strategic approach in outsourcing. Indian software vendors did not simply seek outsourced contract more effectively: they made aggressive strategic in outsourcing. For instance, most software firms of India were initially set up to outsource the contract in western countries, such as United States. By contrary, most Chinese firms seem to position their business in China, a promising yet under-developed market. However, rampant piracy in China took almost 90 percents of potential market, making it impossible for most Chinese firms to obtain sufficient compensation for the investment on development and research, let alone thrive in competitive environment. In the above passage, the author gave an explanation to a particular event, but refuted it soon, until it came with a convincing one – the conclusion.
  • 14. 14 Section 3: Three Subjects Like writing techniques, GMAT subjects may vary significantly. The author may present how caffeine activates human behavior, discuss what causes Japanese auto companies to perform better than those in USA, or explain the union's effort to organize the employees in public sectors. Various as the subjects may be, there are only three major subjects that a GMAT passage may be discussing about: natural science, social science, and business. As the name indicates, natural science topic includes biology, chemistry, geology, and archeology; social science includes art, literature, and civil rights; business includes marketing, advertising, management, and economics. 1. Natural Science Characteristics Most test-takers find the natural science difficult to read and beyond their knowledge base. If you try to figure out what they are really talking about, in most cases you will fail, because reading passages in this kind of subject are filled of nomenclatures and jargons. The good news, however, is that the sentences are always written in a simple syntax and the questions to be answered after reading passage are typical of the recalled questions. That means, when you successfully locate the "key words", you will find it easy to get the right answer. Strategy Don't memorize the details or try to figure out the author's reasoning. Skim the passage, and get its central idea as well as organizational structure. The following passage is about natural science topic. This passage is a little difficult to understand, but the following questions are much easier to answer. Sample Passage The cutting-edge science is ringing alarm bells. Avian flu virus picked up by pigs can swap genetic materials with another flu virus already in the pig and become a new, hitherto unknown flu virus for which no person, no animal has preexisting immunity. The kind of virus causes a pandemic because it spreads from human to human. If you took a peek into history, it turns out that previous influenza pandemics have similar scenarios. The greatest influenza pandemic in 1918 caused more than 20 million deaths of soldiers stationed in France. The last influenza pandemic was in 1968, known as the Hong Kong flu (H3N2). Thousands of deaths and millions were infected worldwide. The other examples are the Nipah virus and Japanese Encephalitis virus, which find pigs to be good hosts. With JE, the virus circulates in the blood of infected pigs. When infected pigs
  • 15. 15 are bitten by Culex mosquitoes, the virus replicates in the mosquito's gut. The next time the mosquito bites a human, the virus is passed on. The pig doesn't get sick as such. The Nipah virus causes pneumonia symptoms in pigs. In humans, it causes encephalitis, and humans catch it only with direct contact with infected pigs. Symptoms range from mild headache to permanent brain damage, and can be fatal. It's merely a phenomenon of nature that the pig is the "mixing vessel" for the new germ. But make no mistake, the pig is not the villain, neither is the chicken. It's actually us, and our horrible farm practices, outdated agricultural policy and, most of all, reckless disregard of our ecology and environment. "Hygiene and management can control what eventually happens," says Lam. "Good farming practice will prevent serious outbreaks and infection to humans." Despite knowing that, animal diseases and the possibility of transmission to humans are becoming quite alarming. Of the 35 new emerging diseases in the last 20 years, more than 70 per cent involved animals. In fact, what we may have done is unwittingly create the perfect launch pad for an influenza pandemic that will likely kill large numbers of people across the globe. Although scientists say it's impossible to predict the odds that the virus will alter its genetic form radically enough to start leaping from human to human, the longer H5N1 is out there killing chickens, the higher the chances are. Sample Question Which of the following statement can be inferred from the passage? (A) New emerging diseases causes more deaths of human than animal. (B) Animals are the villain for most flues. (C) Hygiene and management can not control the spread of viruses. (D) The current bird flu epidemic may be a launch pad for the next influenza pandemic. (E) The influenza pandemic is always a regional phenomenon. Which answer is correct? For choice A, the passage did not make any comparison between deaths of human and deaths of animal. In B, animal is actually not the villain for most flues. Rather, it is human. Look at the second sentence in the fourth paragraph, “But make no mistake, the pig is not the villain, neither is the chicken.” For C, “Hygiene and management can control what eventually happens”(in the middle of fourth paragraph), therefore, C is incorrect. E is also incorrect. Though most flues discussed in this passage were originated from some areas, the passage never stated it was a regional phenomenon. In fact, it “will likely kill large numbers of people across the globe”, as stated at the beginning of last paragraph. The correct answer is D – the current bird flu epidemic may be a launch pad for the next influenza pandemic, because no animal has preexisting immunity and it causes a pandemic by spreading from human to human.
  • 16. 16 2. Social Science Characteristics Why women's rights experienced a significant improvement during 1860's? How the Pullman stroke to improve their living condition? Passages in these subjects are easy to read because it goes as you expect and talks about something around your world. You will find it easy to grasp the main idea and passage map. In order to get the right answer, however, you need to read beyond the words, phrases or concepts in the passage. The right answer is always created in a synthesized way. Strategy Be careful in tackling this "social" passage. To answer the later questions is always not as easy as to understand the passage. The answer choice that contains the exact words or phrases from the passage is generally not the correct answer. Rather, you need to synthesize several sentences or make some reasoning before you pick up the right choice. The process is time-consuming because the social passage is typically long. Sample Passage China as a nation faces two major financial problems. First, eighty-four percent of state-owned enterprises do not generate profit. Government failed to make money from such business. Rather, it has to appropriate substantial funds to these enterprises in order to prevent them from going bankrupt and thus resulting in high unemployment rate. Second, 203 million of civilians in countryside will not be able to gain pension after they retire due to the limited budget of government. I would like to make an outrageous suggestion that would at one stroke generate finance earnings and provide funds for civilians’ retirement. I would propose that government sells its holdings in state-owned enterprises on the open market. Such sales would provide substantial funds for village civilian’s pension. At the same time, they could cut down financial burden on these state-owned enterprises. You might object that government would be deprived of the opportunity to share its enterprise’s profit if someday they make money. I agree. Sell holdings of enterprises that would never generate profit. But, you might reply, every enterprise that competes on the market has potential. Here we part company. Theoretically, you may be correct in claiming that every enterprise has the potential to make money. Practically, you are wrong. I refer to the thousands of state-owned enterprises that are not likely to make money. These companies are 100 percent held by the nation as a whole. Government officials are appointed as the chairman, CEO and president. The management was not responsible for
  • 17. 17 the public interest, but for the nation as a whole. If there is no significant loss in business, they will soon be promoted back to the higher level position in government. If their companies perform great, these executives receive direct money compensation. However, their salary, when combined with such compensation, will be much less than the amount they would earn if were in private company. It would be unrealistic to suggest that village civilians would have sufficient funds if government’s shares were sold on the open market. But the demand for compensating the state-own enterprises would be substantially reduced. Sample Question According to the passage, executives in a state-owned enterprise are motivated by (A) direct money compensation (B) increasing salary (C) political outlook (D) share option (E) social responsibility The passage mentioned the executives of state-owned enterprises in fourth paragraph; therefore, we need not to consider other paragraphs when referring to the original passage. Since “…their salary, when combined with such compensation, will be much less than the amount they would earn if were in private company” as stated in the last sentence, these executives are not motivated by financial earnings. If yes, they will transfer to a private company. Therefore, they are not motivated by direct money compensation, increasing salary, or share option. Rather, they are concerned on their political outlook. “If there is no significant loss in business, they will soon be promoted back to the higher level position in government.” Choice C is the correct answer. For choice E, the passage never discussed the executives’ social responsibility.
  • 18. 18 3. Business Subject Characteristics This subject is highly welcomed since most students possess some knowledge or background in business. But passage of this subject contains the most difficult questions in GMAT Reading Comprehension. Recall questions are few and you always have to reason before you pick up the correct choice. Strategy Don't rely on your memory even if you become or have been quite familiar with its topics. There are too many traps here. Make sure you refer to the passage when answering the questions. Sample Passage The fact that reducing price can generate a competitive advantage for a company does not mean that every reduction in price will create such an advantage. Price reduction, like improvement in service, must be balanced against other types of efforts on the basis of direct, tangible benefits such as increased revenues. If a company is already effectively on a par with its competitors because it provides product at an acceptable price and keeps customers from leaving at an unacceptable rate, then reduction in price may not be effective, since price is not necessarily the deciding factor for any customer in any situation. This truth was not apparent to managers of one operating system software vendor, which failed to improve its competitive position despite its attempt to reduce price. The software managers did not recognize the level of customer inertia that arises from the inconvenience of switching operating system. Nor did they analyze their reduction in price to determine whether it would attract new customers by producing a new standard of price that would excite customers or by proving difficult for competitors to copy. Sample Question The passage suggests which of the following about price charged by an operating system software vendor prior to its strategy in reducing its price? (A) It was slightly low to that of the vendor’s competitors. (B) It threatened to weaken the vendor’s competitive position with respect to other operating system software vendor (C) It had already been reduced after having caused damage to the vendor’s reputation in the past. (D) It enabled the vendor to retain customers at an acceptable rate
  • 19. 19 (E) It needed to be reduced to attain parity with the software provided by competing vendors. Here, the question was created in complicated clauses and itself already hard to understand. In fact, it asks for the situation of the vendor before price reduction. Only D can be inferred from the passage. The original passage stated that “If a company is already effectively on ….. keeps customers from leaving at an unacceptable rate…” and “This truth was not apparent to managers of one operating system software vendor…” That means the vendor was able to retain customers at an acceptable rate.
  • 20. 20 Section 4: Four-step Process of Reading In the previous section we summarize three kinds of subject you will encounter in a GMAT reading passage. Now you will learn the four-step procedure to read a passage in any subject: 1. Analyze the first paragraph. 2. Skim the passage and get some idea of the main idea 3. Identify the purpose of each paragraph and structure of the passage 4. Answer the questions and don't forget to refer to the passage 1. Analyze the first paragraph. It is essential to carefully read the first paragraph. You will get informed what the passage is talking about, and even the main idea of the passage. There are two major reasons for you to carefully read the first paragraph. Fist of all, the paragraph is the main structural unit of any passage. Every paragraph is needed to understand the whole passage or answer the question after the passage. Test-maker never delivers a junk content. It must talk about something that relates to the central idea, and present it as persuasively as possible. In fact, the first paragraph introduces either the position that the author will support or the one that he/she will argue against. So, getting familiar with the introductory paragraph will definitely help you identify the main topic. Secondly, analyzing the fist paragraph in stead of the whole passage can save you much time. As I said at the beginning of this chapter, GMAT reading passage is dry and unfamiliar. It is highly likely that after you read the passage, you get no ideas about what the passage is talking about. If you go back and reread the whole passage, you will have no sufficient time to answer the question. Analyze the first paragraph, pay attention to concepts, and then you will find it easy to understand the subject of passage. Below is the first paragraph of a GMAT reading passage. Pay attention to concept words. China as a nation faces two major financial problems. First, eighty-four percent of state-owned enterprises do not generate profit. Government failed to make money from such business. Rather, it has to appropriate substantial funds to these enterprises in order to prevent them from going bankrupt and thus resulting in high unemployment rate. Second, 203 million of civilians in countryside will not be able to gain pension after they retire due to the limited budget of government. The first sentence stated that China faces two problems. Then, the author specified these two problems using a clear and logical structure. Firstly, government did not make money from but
  • 21. 21 input large amount of money to its enterprises. Secondly, government has limited funds for pension. Now, let's summarize this paragraph and put it in our own words-- China has two problems: financial burden and limited funds. Keep these key words (concepts) in mind, and you will find it easy to understand the remaining passage that we'll present in next step. 2. Skim the passage and get the author's main point Here are some strategies that will speed your reading and help you identify the author's main points: z Focus on the first sentence of each paragraph The first sentence of a paragraph is always the main point of this paragraph. Why? It confirms to the formal writing style. If you are a management consultant, you will find it a great advantage to use a summary at the very beginning of each section. Image when you are presenting a strategy report which contains hundreds of pages, how could your clients catch your all of them? The only solution is to use a highly structured presentation, and summarize your idea at the beginning of each section. In fact, you are also doing like this in the AWA section. By simply reading the first sentence of each paragraph, you can construct a mental road map of the passage while not spending significant time. z Pay attention to the mood words "Mood" words are those that the author uses to demonstrate his/her position to a particular event, phenomenon, or point of view. A mood word can be positive or negative. Positive words such as successfully, correctly and right often illustrate an idea that the author agree. And vis-a-vis, a negative word indicates an idea that will be weakened in later passage. The following sentences express the author's position by using positive mood words: a) Haney's through research provides convincing field evidence that.. b) For many yeas, Benjamin Quarles' seminal account of the participation of African Americans in the American Revolution has remained the standard work in the field. c) Roger Rosenblatt's book successfully alters the approach taken by most previous studies. By contrast, the following mood words are negative. fail ignore overestimate underestimate misunderstand misrepresent overlook exaggerate sound convincingly successfully correctly
  • 22. 22 z Never ignore the counter-evidence indicators The author uses counter-evidence words not to argue against himself, but concede certain minor points that may weaken his argument. The counter evidence is finally refuted by further evidence. You should keep alarm to these words since some students often mistake them as introducing arguing against a statement. Following are some of the most common used counter-evidence indicators: actually despite admittedly except even though nonetheless nevertheless although however In spite of do may OK. Let's go back to the passage talking about national finance. Here are the other five paragraphs. In order for you to skim the passage using the above three techniques, we underlined the first sentences of each paragraph, boldfaced the mood words and italicize the counter-evidence indicators. I would like to make an outrageous suggestion that would at one stroke generate finance earnings and provide funds for civilians’ retirement. I would propose that government sells its holdings in state-owned enterprises on the open market. Such sales would provide substantial funds for village civilian’s pension. At the same time, they could cut down financial burden on these state-owned enterprises. You might object that government would be deprived of the opportunity to share its enterprise’s profit if someday they make money. I agree. Sell holdings of enterprises that would never generate profit. But, you might reply, every enterprise that competes on the market has potential. Here we part company. Theoretically, you may be correct in claiming that every enterprise has the potential to make money. Practically, you are wrong. I refer to the thousands of state-owned enterprises that are not likely to make money. These companies are 100 percent held by the nation as a whole. Government officials are appointed as the chairman, CEO and president. The management was not responsible for the public interest, but for the nation as a whole. If there is no significant loss in business, they will soon be promoted back to the higher level position in government. If their companies perform great, these executives receive direct money compensation. However, their salary, when combined with such compensation, will be much less than the amount they would earn if were in private company.
  • 23. 23 It would be unrealistic to suggest that village civilians would have sufficient funds if government’s shares were sold on the open market. But the demand for compensating the state-own enterprises would be substantially reduced. What is the main idea of the passage? In a word, the author is to present a solution to funding civilian’s pension while benefiting the state-owned enterprises. 3. Diagram the organization of the passage You got main idea of each paragraph. Now, it’s time to ask yourself why the author includes them, what the purpose of each paragraph is, and how each paragraph relates to other. This will help you diagram the organization of a passage, and locate the details when you answer the questions. Pivotal words can help you in diagramming the organization. Pivotal words are signal words or phrases that would in advance indicate the idea of paragraphs. Below represents the most frequently used pivotal words or sentences you will see in a reading passage. Note: A and B represent something, while sb represents somebody. Introduction z When it comes to ..., some think ... z There is a public debate today that ... z A is a common way of ..., but is it a wise one? z Recently the problem has been brought into focus. Presenting Opinion z Now there is a growing awareness that... z It is time we explore the truth of ... z Nowhere in history has the issue been more visible. Further Presenting Opinion z ... but that is only part of the history. z Another equally important aspect is ... z A is but one of the many effects. Another is ... z Besides, other reasons are... Anticipating Objections z You may reply that.
  • 24. 24 z Admittedly, .. z It is reasonable to expect... z It is not surprising that... Exampling z For example(instance),... z ... such as A,B,C and so on (so forth) z A good case in point is... z A particular example for this is... Presenting Reasons z There are many reasons for ... z Why .... , for one thing,... z The answer to this problem involves many factors. z Any discussion about this problem would inevitably involves ... z The first reason can be obliviously seen. z Most people would agree that... z Some people may neglect that in fact ... z Others suggest that... z Part of the explanation is ... Comparing z The advantages for A for outweigh the disadvantages of... z Although A enjoys a distinct advantage ... z Indeed , A carries much weight than B when sth is concerned. z A maybe ... , but it suffers from the disadvantage that... Transitioning z To understand the truth of ..., it is also important to see... z A study of ... will make this point clear Further Anticipating Objections z Certainly, B has its own advantages, such as... z I do not deny that A has its own merits.
  • 25. 25 Conclusion z From what has been discussed above, we may safely draw the conclusion that ... z In summary, it is wiser ... z In short... In step 2, you are assigned to skim the passage and get the main idea. Here, let's identify the purpose of each paragraph for the archeology passage to better understand the passage. (First of all, the author presented the problems) China as a nation faces two major financial problems. First, eighty-four percent of state-owned enterprises do not generate profit. Government failed to make money from such business. Rather, it has to appropriate substantial funds to these enterprises in order to prevent them from going bankrupt and thus resulting in high unemployment rate. Second, 203 million of civilians in countryside will not be able to gain pension after they retire due to the limited budget of government. (Then, the author suggested a solution to the problems) I would like to make an outrageous suggestion that would at one stroke generate finance earnings and provide funds for civilians’ retirement. I would propose that government sells its holdings in state-owned enterprises on the open market. Such sales would provide substantial funds for village civilian’s pension. At the same time, they could cut down financial burden on these state-owned enterprises. (Here, the author anticipated a possible objection) You might object that government would be deprived of the opportunity to share its enterprise’s profit if someday they make money. I agree. Sell holdings of enterprises that would never generate profit. But, you might reply, every enterprise that competes on the market has potential. Here we part company. Theoretically, you may be correct in claiming that every enterprise has the potential to make money. Practically, you are wrong. (Then, the author gave an example to deny this objection) I refer to the thousands of state-owned enterprises that are not likely to make money. These companies are 100 percent held by the nation as a whole. Government officials are appointed as the chairman, CEO and president. The management was not responsible for the public interest, but for the nation as a whole. If there is no significant loss in business, they will soon be promoted back to the higher level position in government. If their companies perform great, these executives receive direct money compensation. However,
  • 26. 26 their salary, when combined with such compensation, will be much less than the amount they would earn if were in private company. (Finally, the author further anticipated a possible objection) It would be unrealistic to suggest that village civilians would have sufficient funds if government’s shares were sold on the open market. But the demand for compensating the state-own enterprises would be substantially reduced. Now, you are able to create a mental road map for the whole passage: Paragraph # 1: introduced two major problems that China faces. Paragraph # 2: suggested a solution and explained why it is effective. Paragraph # 3: anticipated a possible objection and denied it soon. Paragraph # 4: exemplified to argue against a position initiated in the third paragraph. Paragraph # 5: concluded that his solution is not perfect, but really effective By making such a road map, I bet you understand this passage quite well. 4. Tackle the questions and correspondently refer to the passage. Now that you have grasped main idea and the organizational structure of the passage, you are about to answer the following questions. Again, don’t base on your memory. Always refer to the original passage before you pick up a choice. China as a nation faces two major financial problems. First, eighty-four percent of state-owned enterprises do not generate profit. Government failed to make money from such business. Rather, it has to appropriate substantial funds to these enterprises in order to prevent them from going bankrupt and thus resulting in high unemployment rate. Second, 203 million of civilians in countryside will not be able to gain pension after they retire due to the limited budget of government. I would like to make an outrageous suggestion that would at one stroke generate finance earnings and provide funds for civilians’ retirement. I would propose that government sells its holdings in state-owned enterprises on the open market. Such sales would provide substantial funds for village civilian’s pension. At the same time, they could cut down financial burden on these state-owned enterprises. You might object that government would be deprived of the opportunity to share its enterprise’s profit if someday they make money. I agree. Sell holdings of enterprises that would never generate profit. But, you might reply, every enterprise that competes on the
  • 27. 27 market has potential. Here we part company. Theoretically, you may be correct in claiming that every enterprise has the potential to make money. Practically, you are wrong. I refer to the thousands of state-owned enterprises that are not likely to make money. These companies are 100 percent held by the nation as a whole. Government officials are appointed as the chairman, CEO and president. The management was not responsible for the public interest, but for the nation as a whole. If there is no significant loss in business, they will soon be promoted back to the higher level position in government. If their companies perform great, these executives receive direct money compensation. However, their salary, when combined with such compensation, will be much less than the amount they would earn if were in private company. It would be unrealistic to suggest that village civilians would have sufficient funds if government’s shares were sold on the open market. But the demand for compensating the state-own enterprises would be substantially reduced. 1. The primary purpose of the passage is to propose (A) an alternative to manage government property (B) a way to relieve government burden while providing funds to village civilians (C) a way to distinguish state-owned enterprises that make money from those that do no make money (D) the governmental approach to evaluate state-owned enterprise’ executives (E) a new system for national pension system This question requires you to identify the primary concern of the passage as a whole. The first paragraph introduces two major problems that China faces. The second paragraph suggests a solution and explains why it is effective. The third anticipates a possible objection and refutes it soon. The fourth paragraph illustrates an example to support the author’s argument. In the last paragraph, the author concludes that his solution is not perfect, but really effective. Therefore, the correct answer is B. 2. The author implies that all of the following statements about enterprises with which government holds 100 percent share are true EXCEPT: (A) A market for government’s share already exists. (B) Such enterprises seldom generate profit. (C) There is likely to be a continuing loss of such enterprises. (D) Government officers are appointed as the executives with such enterprises.
  • 28. 28 (E) If the executives perform poorly, they will be demoted to lower position. The question requires you to identify the answer choice that CANNOT be inferred from the passage. Nothing in the passage implies that “the executives will be demoted to lower position if they perform poorly”. Therefore, the best answer is E. In answering the question that contains “EXCEPT”, keep alarm not to be fooled by the test maker. 3. The author implies that which of the following would occur if government’s shares were sold on the open market? I. The shortage of retirement fund in village would eventually cease completely. II. Current executives in state-owned enterprises are not motivated to perform better III. Civilians in countryside would be able to seek sufficient funds from government. (A) I only (B) II only (C) I and II only (D) II and III only (E) I, II, and III This question asks you to identify information that is suggested rather than directly stated in the passage. To answer it, first look for the location in the passage of the information specified in answer choice. The last paragraph states that “It would be unrealistic to suggest that village civilians would have sufficient funds if government’s shares were sold on the open market”, therefore, I is incorrect. III, which is a repeated of I, is also incorrect. Only II can be inferred from the original passage, therefore B is the best answer.
  • 29. 29 Section 5: Five Types of Questions While the techniques introduced in previous four sections speed your reading, this section is developed to help you pick up the right choice quickly and decisively. In the following passage, we will discuss the major question types you may encounter in real GMAT test. Generally, there are only five major types of questions. As you become familiar with the following question types, you will gain an intuitive sense for the places from which questions are likely to be drawn. Note, the order in which the questions are asked roughly corresponds to the order in which the main issues are presented in the passage. Early questions should correspond to information given early in the passage, and so on. Of course, there are many other kinds of classification according to different criteria. Here, we classify, by how we solve reading comprehension questions, into five based on the summary of thousands of the previous real questions. Let's preview the five question types. Question Types Preview 1. Main Idea Question a) Main Topic b) Tone c) Structure d) Exemplifying 2. Recall Question a) Description b) Listing 3. Inference Question 4. Critical Reasoning Question a) Analogy b) Assumption/Weaken/Strengthen 5. Unable-to-locate Question
  • 30. 30 1. Main Idea Question There are four sub-types for this kind of question: Main Topic, Tone, Structure, and Exemplifying. Why should we incorporate them into one type of question? In answering Main Idea Question, you should understand the organizational structure of the passage, the author tone toward a particular point of view in the reading passage, the purpose of each paragraph and why a particular example was illustrated. In other words, if you can determine the main topic of the passage, you are simultaneously well informed with the structure, the intent of specific example, and tone toward specific position. A. Main Topic Main idea questions test your ability to identify and understand an author's intent in a passage. The main idea is usually stated in the first or last paragraph. Main idea questions are usually the first questions asked. Some common main idea questions include: z The primary purpose of the passage is to… z Which of the following titles would best describe the content of the passage? z The passage supplies information that would answer which of the following questions? z Which of the following is the principal topic of the passage? z The passage is most probably an excerpt from. z Which of the following best states the central idea of the passage? In most cases, main idea questions are easy to solve. In most GMAT passage the author's primary purpose is to persuade the reader to accept her opinion. Occasionally, it is to describe something. By determining the relationship of each paragraph, you come up with the main ideal at the same time. However, the GMAT writers may obscure the correct answer by surrounding it with close answer choices that stress specifics. Eliminate these choices without hesitation on the test day. Trap 1: The main topic will not focus on certain details in the passage. If you encounter the main ideal question, eliminate the answer choices that describe the details. Trap 2: Pay special attention to the "repeat" answer. Certain choices may exactly repeat some or most words of the correct answer, but do not present the central idea, therefore, is not the correct answer. B. Tone Question Tone questions ask you to identify the writer's attitude. Is the writer's feeling toward the subject positive, negative, or neutral? The following represents some ways of the questions asked.
  • 31. 31 z Which of the following best summarizes the author's evaluation of Bailyn's fourth proposition? z The author's attitude toward the culture in most factories is best described as (A) cautious (B) critical (C) disinterested (D) respectful (E) adulatory However, if you did not get a feel for the writer's attitude on the first reading, check the mood words that he chooses. Beware of answer choices that contain extreme emotions. Remember the passages are taken from academic journals. In the rarefied air of academic circles, strong emotions are considered inappropriate and sophomoric. The writers want to display opinions that are considered and reasonable, not spontaneous and off-the-wall. So if an author's tone is negative, it may be disapproving, not snide or ridiculous. If her tone is positive, it may be approving, not ecstatic. Or if her tone is neutral, it would be not be disinterested. C. Organizational Structure When you can determine the right answer for main topic, generally you have been familiar with the organizational structure. Every passage is consisted of some paragraphs, and each single paragraph performs some certain function to the passage as a whole, by presenting, supporting or refuting the central idea. So, think about the purpose of each paragraph as you read through the passage. You may encounter one type of question concerning the main idea or purpose of some certain paragraph. Some common questions include: z The last paragraph of the passage performs which of the following functions? z Which of the following best describes the organization of the second paragraph? z Which of the following best describes the organization of the passage? z Which of the following best describes the relation of the first paragraph to the passage as a whole? It is relatively easy to solve this kind of question for two reasons. First, however the question may ask, it is concerning the main idea of the paragraph. If you come up with a question including "paragraph", it definitely require you to generate that paragraph. Second, paragraph is only some element of the completed passage. If you can generate the main topic for the whole passage of three to four paragraphs, why aren't you able to summarize just one paragraph? D. Exemplifying The other type of structure question, exemplifying, tests your ability to identify the intention of author's illustrating of something, some people, or phenomenon. In answering this question, you need to first locate the example, and then refer to opinion preceding or accompanying the example. The right answer is the repeat of this opinion. z In illustrating the example in line 13-16, the author intended to.?
  • 32. 32 z The author referred to the experiment in order to.? In addition to the above four types, the Main Idea Question may require you to respond in other ways, such as: z From what kinds of subject could this passage be excerpted? z Which of the following topic would be preceding this passage? This question, however, is similar to the above four, since they are based on your understanding of the completed passage. Sample Question #1 The cutting-edge science is ringing alarm bells. Avian flu virus picked up by pigs can swap genetic materials with another flu virus already in the pig and become a new, hitherto unknown flu virus for which no person, no animal has preexisting immunity. The kind of virus causes a pandemic because it spreads from human to human. If you took a peek into history, it turns out that previous influenza pandemics have similar scenarios. The greatest influenza pandemic in 1918 caused more than 20 million deaths of soldiers stationed in France. The last influenza pandemic was in 1968, known as the Hong Kong flu (H3N2). Thousands of deaths and millions were infected worldwide. The other examples are the Nipah virus and Japanese Encephalitis virus, which find pigs to be good hosts. With JE, the virus circulates in the blood of infected pigs. When infected pigs are bitten by Culex mosquitoes, the virus replicates in the mosquito's gut. The next time the mosquito bites a human, the virus is passed on. The pig doesn't get sick as such. The Nipah virus causes pneumonia symptoms in pigs. In humans, it causes encephalitis, and humans catch it only with direct contact with infected pigs. Symptoms range from mild headache to permanent brain damage, and can be fatal. It's merely a phenomenon of nature that the pig is the "mixing vessel" for the new germ. But make no mistake, the pig is not the villain, neither is the chicken. It's actually us, and our horrible farm practices, outdated agricultural policy and, most of all, reckless disregard of our ecology and environment. "Hygiene and management can control what eventually happens," says Lam. "Good farming practice will prevent serious outbreaks and infection to humans." Despite knowing that, animal diseases and the possibility of transmission to humans are becoming quite alarming. Of the 35 new emerging diseases in the last 20 years, more than 70 per cent involved animals. In fact, what we may have done is unwittingly create the perfect launch pad for an influenza pandemic that will likely kill large numbers of people across the globe. Although scientists say it's impossible to predict the odds that the virus will alter its genetic form radically enough
  • 33. 33 to start leaping from human to human, the longer H5N1 is out there killing chickens, the higher the chances are. Which of the following best describes the topic of the passage? (A) What causes the Nipah virus and Japanese Encephalitis virus to happen? (B) Does Hong Kong flu originate from pig? (C) From fowl to pigs to humans? (D) Is influenza pandemic horrible? (E) Shall we eat chicken? This question asks you to find a title for the passage. In other word, it requires you to identify the primary concern of the passage as a whole. The first paragraph presents a recent virus. The second and third paragraphs describe similar influenza pandemics in history. The fourth paragraph concludes who should be responsible for the spread of virus and what human can do to control. The last paragraph indicates that people stimulated rather than inhibited its promulgation. We can thus conclude the current virus will also leap to human. Furthermore, the passage as a whole is to “ring alarm bells”. Therefore, C is the best answer. Sample Question #2 Indian firms have achieved the highest levels of efficiency in the world software outsourcing industry. Some researchers have assumed that Indian firms use the same programming languages and techniques as Chinese firms but have benefited from their familiarity with English, the language used to write software code. However, if this were true, then one would expect software vendors in Hong Kong, where most people speak English, to perform not worse than do Indian vendors. However, this is obviously not the case. Other researchers link high Indian productivity to higher levels of human resource investment per engineer. But a historical perspective leads to a different conclusion. When the two top Indian vendors matched and then doubled Chinese productivity levels in the mid-eighties, human resource investment per employee was comparable to that of Chinese vendors. Furthermore, by the late eighties, the amount of fixed assets required to develop one software package was roughly equivalent in India and in the China. Since human resource investment was not higher in India, it had to be other factors that led to higher productivity. A more fruitful explanation may lie with Indian strategic approach in outsourcing. Indian software vendors did not simply seek outsourced contract more effectively: they made aggressive strategic in outsourcing. For instance, most software firms of India were initially set up to outsource the contract in western countries, such as United States. By contrary, most Chinese firms seem to position their business in China, a promising yet
  • 34. 34 under-developed market. However, rampant piracy in China took almost 90 percents of potential market, making it impossible for most Chinese firms to obtain sufficient compensation for the investment on development and research, let alone thrive in competitive environment. Which of the following best describes the organization of the first paragraph? (A) A thesis is presented and supporting examples are provided. (B) Opposing views are presented, classified, and then reconciled. (C) A fact is stated, and an explanation is advanced and then refuted. (D) A theory is proposed, considered, and then amended. (E) An opinion is presented, qualified, and then reaffirmed. This question requires you to identify the organizational structure of the first paragraph. In this paragraph, the author first states a fact that Indian firms achieved the highest efficiency in software outsourcing. Then, an assumption is presented to explain such phenomenon. However, the author refuted this explanation soon. Thus, C is the best answer.
  • 35. 35 2. Recall Question There are two subtypes of recall questions: detail-locating and listing. In the following passage, we'll discuss one by one. A. Detail-locating Locating question is the most common question you will encounter in Reading Comprehension. It roughly constitutes to 50-60% of total numbers of questions. That means, in every reading passage, there will be about one or two detail-locating questions. It is quite simple, however, to solve this seemingly difficult question if you are able locate the detail tested. The right answer choice is rewritten from certain sentence in the passage by changing some words or phrases. For example, test writer will change some words from adjective to adverbial, from noun to gerund, or just change to its synonym. Strategy: How to locate Below we will introduce the three-step method to locate detail. (1) Before you locate the question to passage, you need to determine what to locate. Key words are something that is mentioned both in the question and in passage. Then, what are key words? Look at the following question: Which of the following is mentioned in the passage as a disadvantage of storing artifacts in museum basements? Here, key words will not be any word or phrase of "which of the following is mentioned in the passage as", but will be "disadvantage" from "disadvantage of storing artifacts in museum basements". We call this step as Defining Key Words. (2) After you define key words, you are turning to the original passage. Sometimes, the key words will appear several times in different parts of the passage. Where should you refer to? Generally, you should locate the key words to the sentence in which key words first appear. After all, you have only several minutes to complete a passage. (3) When you determined which sentence (sometimes, two or more sentences) to locate, get some idea, then quickly refer to the answer choices. Do not spend too much time analyzing this sentence since it may be too long or complicated to understand. If the choice mentions something that only appears in other part of the passage, eliminate it. Also eliminate the choice that just repeats the words or phrases from original passage. B. Listing The other type of Recall Question is listing. As the name indicates, Listing Question requires you to identify some people, actions, or situations that are enumerated in the passage.
  • 36. 36 Here are some Listing Questions: z According to the passage, senior managers use intuition in all of the following ways EXCEPT to: z According to the passage, critics of the Ewha women's studies program cited the program as a threat to which of the following? I. National identity II. National unification III. Economic development IV. Family integrity (A) I only (B) I and II only (C) I, II, and III only (D) II, III, and IV only (E) I, II, III, and IV To solve this type of question, you should first name the key word from the stimulus, and locate it to original passage. Then, you will find some lists that are similar to the answer choices. Carefully compare those lists one by one to the answer choices. Use POE to eliminate incorrect choice, until you find the right one.
  • 37. 37 3. Inference Questions Inference question is the second most common. Unlike recall question, inference questions require you to go beyond the passage. That means, the correct answer must say more than what is said in the passage. Beware of same language traps with these questions: the correct answer will often both paraphrase and extend a statement in the passage, but it will not directly quote it. If you are puzzled how to determine whether a detail question is recall question or inference question, pay attentions to the way the question asks. Generally, inference question will include some word, such as infer, suggest and imply that indicates what kind of question it is. z It can be inferred from the passage that... z The passage/author suggests that. z The passage/author implies that... Since we must not directly refer to the original passage in answering inference question, we need to decipher the inference. Next, we will show you how to reason from couples of sentence. Technique 1 Reasoning by Word of Comparison The question is asking about B, but you may be unable to directly identify the characters of B even you have located B. Rather, the original sentence is discussing about A. Here, you should turn to the word that indicates comparison between A and B. Some words that indicate strong comparison are unlike, in contrast to, by contrast and compared with. When you can determine the character of B, you can simultaneously determine A is B or non-B. Also, the passage may compare two particular events by dates or places. The phrases could be "prior to 1975" or "since mid-1970's". Technique 2 Reasoning by Syllogism In logics, Syllogism looks like this: every virtue is laudable; kindness is a virtue; therefore, kindness is laudable. As we put it in more simple way, it may be "A→B and, then A→C". It may be relatively easy to recognize A→B by locating the key word in the question, but it will always take some time to identify B→C, since they may be located in other part of the place. So pay attention to the pronouns (it or they) and the nouns with definite article "the" since they often serve as B. The fact that reducing price can generate a competitive advantage for a company does not mean that every reduction in price will create such an advantage. Price reduction, like improvement in service, must be balanced against other types of efforts on the basis of direct, tangible benefits such as increased revenues. If a company is already effectively on a par with its competitors because it provides product at an acceptable price and keeps customers from leaving at an unacceptable rate, then reduction in price may not be effective, since price is not necessarily the deciding factor for any customer in any situation.
  • 38. 38 This truth was not apparent to managers of one operating system software vendor, which failed to improve its competitive position despite its attempt to reduce price. The software managers did not recognize the level of customer inertia that arises from the inconvenience of switching operating system. Nor did they analyze their reduction in price to determine whether it would attract new customers by producing a new standard of price that would excite customers or by proving difficult for competitors to copy. The passage suggests which of the following about price charged by an operating system software vendor prior to its strategy in reducing its price? (A) It enabled the vendor to retain customers at an acceptable rate (B) It threatened to weaken the vendor’s competitive position with respect to other operating system software vendor (C) It had already been reduced after having caused damage to the vendor’s reputation in the past. (D) It was slightly low to that of the vendor’s competitors. (E) It needed to be reduced to attain parity with the software provided by competing vendors. Here, the question asks for the situation of the vendor before price reduction. Only A can be inferred from the passage. The original passage stated that “If a company is already effectively on ….. keeps customers from leaving at an unacceptable rate…” and “This truth was not apparent to managers of one operating system software vendor…” That means the vendor was able to retain customers at an acceptable rate.
  • 39. 39 4. Critical Reasoning Question Even in reading comprehension, you will encounter some critical reasoning questions: analogy, assumption, weaken, and strengthen. Here, the whole passage is an argument with premises, assumptions and conclusions. The question asks you to identify the reasoning, critique the argument or recognize the potential assumption. When you need to do is also to first locate the conclusion to particular sentence of the passage, then identify the evidence and conclusion. The premise (or evidence) could be near to the conclusion, or in other part of the 3-4 paragraph passage. So, it is more difficult than its counterpart in critical reasoning section. That's why it appears more often in high difficult level screen. A. Analogy Also known as application question, analogy question requires you to identify the author's reasoning somewhere in the passage and then ask you to select one from the following five answer choices that reasons as that is presented in the passage. The answer choices are generally long and complicated, but they are not so difficult to understand. After you locate the details to certain sentences in the passage, try to identify the reasoning, and then turn to the answer choices. Fortunately, once you identify the reasoning, you will quickly get the right answer since there are great differences among these five choices. B. Assumption/Weaken/Strengthen Weaken, Support, and Assumption are the other three types of question you are expected to solve in critical reasoning question. As we said above, you need to evaluate the argument and identify the assumptions. Typical questions would be: z Which of the following, if true, would most weaken the theory proposed by Snyder et al? z Which of the following, if true, would most strongly support Keyssar's findings as they are described by the author? Sample Question The cutting-edge science is ringing alarm bells. Avian flu virus picked up by pigs can swap genetic materials with another flu virus already in the pig and become a new, hitherto unknown flu virus for which no person, no animal has preexisting immunity. The kind of virus causes a pandemic because it spreads from human to human. If you took a peek into history, it turns out that previous influenza pandemics have similar scenarios. The greatest influenza pandemic in 1918 caused more than 20 million deaths of soldiers stationed in France. The last influenza pandemic was in 1968, known as the Hong Kong flu (H3N2). Thousands of deaths and millions were infected worldwide.
  • 40. 40 The other examples are the Nipah virus and Japanese Encephalitis virus, which find pigs to be good hosts. With JE, the virus circulates in the blood of infected pigs. When infected pigs are bitten by Culex mosquitoes, the virus replicates in the mosquito's gut. The next time the mosquito bites a human, the virus is passed on. The pig doesn't get sick as such. The Nipah virus causes pneumonia symptoms in pigs. In humans, it causes encephalitis, and humans catch it only with direct contact with infected pigs. Symptoms range from mild headache to permanent brain damage, and can be fatal. It's merely a phenomenon of nature that the pig is the "mixing vessel" for the new germ. But make no mistake, the pig is not the villain, neither is the chicken. It's actually us, and our horrible farm practices, outdated agricultural policy and, most of all, reckless disregard of our ecology and environment. "Hygiene and management can control what eventually happens," says Lam. "Good farming practice will prevent serious outbreaks and infection to humans." Despite knowing that, animal diseases and the possibility of transmission to humans are becoming quite alarming. Of the 35 new emerging diseases in the last 20 years, more than 70 per cent involved animals. In fact, what we may have done is unwittingly create the perfect launch pad for an influenza pandemic that will likely kill large numbers of people across the globe. Although scientists say it's impossible to predict the odds that the virus will alter its genetic form radically enough to start leaping from human to human, the longer H5N1 is out there killing chickens, the higher the chances are. All of the following situations are similar to the spread of avian flu virus described in the first paragraph EXCEPT: (A) The BT2 spread from a pig to another pig, and thus causes significant disease in pig. (B) The AIDS viruses transferred from monkeys to man and spread across the world. (C) The SARS virus originates from some wildlife and is picked up by civet cats from which humans got it. (D) Nipah virus circulates in the blood of infected pig, which is bitten by Culex mosquitoes, the virus replicates in the mosquito's gut. The next time the mosquito bites a human, the virus is passed on. (E) H5N1 starts in chickens and leaps from human to human. The question requires you to recognize a situation that is not similar to the spear of avian flu. Before considering following answer choices, we fist define its rationale. It is something like this: Avian flu virus picked up by pigs and is transferred to human. All of the situations described in the answer choices are similar to it ex that in choice A (from animal to animal). Therefore, A is the best answer.
  • 41. 41 5. Difficult-to-locate Question Some question does not ask for the central idea of a passage. Rather, it requires you to draw a conclusion based on the passage: z According to the passage, which of the following is the author most likely to agree with? z The passage supplies information that would answer which of the following questions? Unlike Recall Question or Inference Question, Difficult-to-locate Question does not contain key words that you can use to locate the details tested. In order to solve this type of question, you have to skim through the passage again and again until you get the right answer. Eliminating wrong choices often take considerable time since the answer choices are often too long and complicated to understand. That is why most test takers regard difficult-to-locate question as the most difficult one in reading comprehension. The good news is that if you encounter several questions like these, then you probably get a high score since questions are presented based on your performance on the previous questions. Sample Question Indian firms have achieved the highest levels of efficiency in the world software outsourcing industry. Some researchers have assumed that Indian firms use the same programming languages and techniques as Chinese firms but have benefited from their familiarity with English, the language used to write software code. However, if this were true, then one would expect software vendors in Hong Kong, where most people speak English, to perform not worse than do Indian vendors. However, this is obviously not the case. Other researchers link high Indian productivity to higher levels of human resource investment per engineer. But a historical perspective leads to a different conclusion. When the two top Indian vendors matched and then doubled Chinese productivity levels in the mid-eighties, human resource investment per employee was comparable to that of Chinese vendors. Furthermore, by the late eighties, the amount of fixed assets required to develop one software package was roughly equivalent in India and in the China. Since human resource investment was not higher in India, it had to be other factors that led to higher productivity. A more fruitful explanation may lie with Indian strategic approach in outsourcing. Indian software vendors did not simply seek outsourced contract more effectively: they made aggressive strategic in outsourcing. For instance, most software firms of India were initially set up to outsource the contract in western countries, such as United States. By contrary, most Chinese firms seem to position their business in China, a promising yet under-developed market. However, rampant piracy in China took almost 90 percents of potential market, making it impossible for most Chinese firms to obtain sufficient
  • 42. 42 compensation for the investment on development and research, let alone thrive in competitive environment. According to the passage, which of the following statements is true of Indian software developers? (A) Their productivity levels did not equal those of Chinese software engineers until the late eighties. (B) Their high efficiency levels are a direct result of English language familiarity. (C) They develop component-specific software. (D) They are built to outsource the western orders. (E) They develop more packages of software than do those in Chinese developers. In the middle of the last paragraph, the author states that “For instance, most software firms of India were initially set up to outsource the contract in western countries, such as United States.” Thus, the best answer is D.
  • 43. 43 Section 6: Six test points While four-step procedure helps you to understand a passage and the five types of question guide you how ETS test the understanding of the passage, the six test points will in advance introduce what would be tested even before you read the questions. As you are reading the passage, keep alarm to certain words or phrases since they would later act as clues for answering the following questions. We call these signal words or phrases as test points. In the following passage, we will introduce you the six most common test points in reading comprehension. Once you become familiar with these test points, you will get advantage in speed to come up with the right answer choice. 1. Comparison Words or phrases: like, unlike, in contrast to, similarly Question Type: recall question, inference question Here is an example: The fact that reducing price can generate a competitive advantage for a company does not mean that every reduction in price will create such an advantage. Price reduction, like improvement in service, must be balanced against other types of efforts on the basis of direct, tangible benefits such as increased revenues. If a company is already effectively on a par with its competitors because it provides product at an acceptable price and keeps customers from leaving at an unacceptable rate, then reduction in price may not be effective, since price is not necessarily the deciding factor for any customer in any situation. This truth was not apparent to managers of one operating system software vendor, which failed to improve its competitive position despite its attempt to reduce price. The software managers did not recognize the level of customer inertia that arises from the inconvenience of switching operating system. Nor did they analyze their reduction in price to determine whether it would attract new customers by producing a new standard of price that would excite customers or by proving difficult for competitors to copy. Sample question According to the passage, reduction in price are comparable to improvement in service in terms of the (A) tangibility of the benefits that they tend to confer (B) increased revenues that they ultimately produce
  • 44. 44 (C) basis on which they need to be weighed (D) insufficient analysis that managers devote to them (E) degree of competitive advantage that they are likely to provide To answer this question, first locate the question to the second sentence of the passage. "Price reduction, like improvement in service, must be balanced against other types of efforts on the basis of direct, tangible benefits such as increased revenues." In other words, they are comparable based on which they need to be weighed. Therefore, C is the correct answer.
  • 45. 45 2. Example & Listing Words or phrase: such as, as well as, for example, for instance Question type: Listing, Exampling Let's look at a sample question for the same passage. The fact that reducing price can generate a competitive advantage for a company does not mean that every reduction in price will create such an advantage. Price reduction, like improvement in service, must be balanced against other types of efforts on the basis of direct, tangible benefits such as increased revenues. If a company is already effectively on a par with its competitors because it provides product at an acceptable price and keeps customers from leaving at an unacceptable rate, then reduction in price may not be effective, since price is not necessarily the deciding factor for any customer in any situation. This truth was not apparent to managers of one operating system software vendor, which failed to improve its competitive position despite its attempt to reduce price. The software managers did not recognize the level of customer inertia that arises from the inconvenience of switching operating system. Nor did they analyze their reduction in price to determine whether it would attract new customers by producing a new standard of price that would excite customers or by proving difficult for competitors to copy. The discussion of the operating system software vendor last paragraph serves which of the following functions within the passage as a whole? (A) It describes an exceptional case in which reduction in price actually failed to produce a competitive advantage. (B) It illustrates the pitfalls of choosing to reduce price at a time when business strategy is needed more urgently in another area. (C) It demonstrates the kind of analysis that managers apply when they choose one kind of business strategy over another (D) It supports the argument that strategies in certain aspects are more advantageous than strategies in other aspects. (E) It provides an example of the point about reduction in price made in the first paragraph. Clearly, the author intends to prove his position that reduction in price does not necessarily generate competitive advantage. E is the correct.
  • 46. 46 3. People, Date & Place Phrase: in the nineteenth-century, prior to mid-1970's, Snyder proposed that. Question: inference question, main idea question Indian firms have achieved the highest levels of efficiency in the world software outsourcing industry. Some researchers have assumed that Indian firms use the same programming languages and techniques as Chinese firms but have benefited from their familiarity with English, the language used to write software code. However, if this were true, then one would expect software vendors in Hong Kong, where most people speak English, to perform not worse than do Indian vendors. However, this is obviously not the case. Other researchers link high Indian productivity to higher levels of human resource investment per engineer. But a historical perspective leads to a different conclusion. When the two top Indian vendors matched and then doubled Chinese productivity levels in the mid-eighties, human resource investment per employee was comparable to that of Chinese vendors. Furthermore, by the late eighties, the amount of fixed assets required to develop one software package was roughly equivalent in India and in the China. Since human resource investment was not higher in India, it had to be other factors that led to higher productivity. A more fruitful explanation may lie with Indian strategic approach in outsourcing. Indian software vendors did not simply seek outsourced contract more effectively: they made aggressive strategic in outsourcing. For instance, most software firms of India were initially set up to outsource the contract in western countries, such as United States. By contrary, most Chinese firms seem to position their business in China, a promising yet under-developed market. However, rampant piracy in China took almost 90 percents of potential market, making it impossible for most Chinese firms to obtain sufficient compensation for the investment on development and research, let alone thrive in competitive environment. The author suggests that if the researchers of India mentioned in paragraph 1 were correct, which of the following would be the case? (A) The computer used in India software firms would be different from the computer used in China firms. (B) Indian engineers would be trained to do several different programming jobs. (C) Familiarity with English language would not have an influence on the productivity levels of engineers. (D) The engineers in India-run firms would have lower productivity levels if they have a poor command of English.
  • 47. 47 (E) The production levels of India-run firms located in the China would be equal to those of firms run by China firms. If the researchers are correct, then the familiarity with English determines the productivity of engineers. That is, if the engineers in India-run firms have a poor command of English they would have lower productivity levels, as stated in choice D.